Космос и астрономия


Ответить в тред Ответить в тред

Check this out!
<<
Назад | Вниз | Каталог | Обновить тред | Автообновление
558 27 104

Тред тупых вопросов №115 Schwarzschild edition Аноним 25/10/19 Птн 06:24:12 5247971
KarlSchwarzschi[...].jpg (12Кб, 206x300)
206x300
Black Hole.jpg (33Кб, 540x392)
540x392
the-blackhole-o[...].jpg (44Кб, 638x479)
638x479
14191629.mp4 (9695Кб, 1280x720, 00:00:39)
1280x720
Тред вопросов о жизни, Вселенной и всём таком.

Спрашиваем то, за что в других местах выдают путёвку в биореактор. Здесь анонимные учёные мирового уровня критически рассмотрят любые гениальные идеи и нарисованные в Paint схемы.

Предыдущий тут: >>519323 (OP)
https://2ch.hk/spc/res/519323.html

Q: Можно быстрее?
A: Можно упасть в Пузырь Алькубьерре, NASA уже почти надула его (это шутка).

Q: Я начитался охуительных историй про уфологию, че делать, нам жопа?
A: Да, тебе жопа, можешь сгонять в зогач или куда оттуда пошлют.

Q: Что будет с человеком в вакууме без скафандра / если он упадет на черную дыру / попробует ступить на поверхность газового гиганта/солнца?
A: Он умрёт.

Q: Почему бы не привязать ракету к воздушному шару или стартовать с горы?
A: Космос - это не как высоко, а как быстро, большая часть энергии ракеты уходит на разгон вбок.
Подробнее тут https://what-if.xkcd.com/58/ (английский) https://chtoes.li/orbital-speed/ (перевод)
Аноним 25/10/19 Птн 06:26:08 5247982
>>524797 (OP)
Как узнали, что внутри черной дыры есть сингулярность?
Аноним 25/10/19 Птн 06:32:17 5248013
>>524798
По математике так получилось, под горизонтом ничего не может двигаться в направлении изнутри наружу и вся материя должна упасть в центральную точку (или кольцо) за конечное время.

Если все действительно так, то образование сингулярности нулевого объема и бесконечной плотности — неизбежно.
Аноним 25/10/19 Птн 06:34:05 5248024
>>524801
>По математике так получилось
А может математически ошиблись и там нет никакой сингулярности?
Аноним 25/10/19 Птн 06:41:33 5248045
>>524802
Предложи физику получше.
Аноним 25/10/19 Птн 06:56:16 5248076
>>524804
Получше? Что значит получше? Никто не знает что внутри черной дыры и никогда это не узнает.
Аноним 25/10/19 Птн 06:56:29 5248087
>>524797 (OP)
Ебанный стыд...
Во-первых, Алькубьерре.
Во-вторых, не упасть, а создавать вокруг корабля изнутри (иначе кина не будет).
В-третьих, НАСА искривляет пространство на десятимиллионную часть, контролируя это сверхточными интерферометрами, до самого варп-привода здесь - как до Антарктиды раком.
Аноним 25/10/19 Птн 07:06:48 5248098
1312424.jpg (83Кб, 600x800)
600x800
>>524808
Пузырь Алькубьерре — идея, основанная на решении уравнений Эйнштейна, предложенная мексиканским физиком-теоретиком Мигелем Алькубьерре.
Аноним 25/10/19 Птн 07:30:51 5248119
Аноним 25/10/19 Птн 07:31:47 52481210
Аноним 25/10/19 Птн 07:50:55 52481311
Эскобарке.
Аноним 25/10/19 Птн 08:35:14 52482412
Аноним 25/10/19 Птн 08:51:11 52483013
Аноним 25/10/19 Птн 09:18:04 52483514
>524754
> Червоточины - вообще один из самых очевидных квантовогравитационных эффектов
Чел, ты сейчас говоришь о несуществующих вещах так, как будто они существуют и все уже доказано.
Ну хорошо, допустим квантовая гравитация есть. Как ты будешь открывать червоточину? Как ты откроешь ее в нужном месте и напраалении? Как ты будешь делать ее стабильной, чтобы можно было успеть пролететь? Как ты вообще сквозь нее будешь летать?
> Ты удивишься, но в рамках одного из современных подходов к решению информационного парадокса физики черных дыр кванты хокинговского излучения можно интерпретировать как своего рода выходы червоточин, ведущих из-под горизонта черной дыры наружу.
Это недоказанные гипотезы, основанные на других недоказанных гипотезах, лол.

Чел, это все очень зыбко, неубедительно и не имеет под собой жесткого фундамента, но ты говоришь об этом с такой уверенностью. Мдаа, прям как фанатик какой то.
Аноним 25/10/19 Птн 09:39:52 52483715
>>524812
>Ебанный
Вот уж действительно стыд
Аноним 25/10/19 Птн 09:48:46 52483916
>>524835
>Как
Каком кверху. Вот когда откроют квантовую теорию гравитации, тогда и поговорим. Сейчас речь идет лишь о принципиальной возможности червоточин и о том, что нет принципиальных препятствий к их существованию.
>неубедительно
Лавр Федотыч, вы? Фундамент там жестче не придумаешь математика, другое дело, имеет ли все это хоть какое-то отношение к реальности. Вот еще хуимболка, где черные дыры натурально полагаются бозе-эйнштейновским конденсатом гравитонов:
https://arxiv.org/pdf/1212.0765
Если что, это "менее надежная" теория, чем червоточины из черных дыр.
>прям как фанатик
Ага, скажи еще, что я адепт научной неорелигии квантования гравитации. Миша, ты? На самом деле я просто хотя бы немного знаю о чем говорю, в отличие от тебя кек.
Аноним 25/10/19 Птн 10:12:05 52484417
Пузырь Хуевертке.
Аноним 25/10/19 Птн 11:33:17 52486218
>>524801
>упасть в центральную точку (или кольцо
А чому бы не тело некоторого конечного размера?
Аноним 25/10/19 Птн 11:56:00 52486419
>>524835
>Червоточины - вообще один из самых очевидных квантовогравитационных эффектов

Очевидных? Кто наблюдал в реальности хоть что-то отдаленно на это похожее? Это блядь ебаная "магия" и она нихуя не очевидна.
Аноним 25/10/19 Птн 12:29:18 52486720
>>524862
Какие силы удержат его от дальнейшего сжатия? Имей в виду, что сильное, слабое и электромагнитное взаимодействие внутри дыры не работают в направлении сингулярность → горизонт, потому что их переносчики (фотон, глюоны, W- и Z- бозоны) тоже движутся со скоростью света и не могут подняться выше из гравитационного колодца, это требует сверхсветовой скорости.
Аноним 25/10/19 Птн 12:38:47 52487021
>>524801
Дело в том что математика != физика. Математические модели они часто не полны, что-то не учитывают. Поэтому чорних дырок нет, вещество скорее всего не может так сильно сжиматься.
Аноним 25/10/19 Птн 12:41:53 52487222
>>524870
>чорних дырок нет
Прямое наблюдение уже не считается?
Аноним 25/10/19 Птн 12:42:47 52487323
>>524872
Не было прямого наблюдения.
Аноним 25/10/19 Птн 12:44:49 52487524
>>524873
А что видели радиотелескопы? Интерпретируй в рамках консервативной теории тень горизонта событий.
Аноним 25/10/19 Птн 12:46:25 52487625
Аноним 25/10/19 Птн 12:46:44 52487726
>>524867
>Какие силы удержат его от дальнейшего сжатия?
Мб какие-то квантовые силы?
Аноним 25/10/19 Птн 12:48:33 52487927
>>524875
Они видели гранты с большим количеством ноликов.
Аноним 25/10/19 Птн 12:49:55 52488028
>>524877
>квантовые силы
Жидовские выдумки не нужны.
Аноним 25/10/19 Птн 12:50:20 52488129
1200px-Blackhol[...].jpg (31Кб, 1200x1200)
1200x1200
>>524876
Большое ничего.
Чем тогда является невидимый центр масс в центре нашей Галактики? Миллионы масс Солнца, ничего не излучает.
Аноним 25/10/19 Птн 12:50:55 52488230
Поясните, это толстота или очередной набег зогошизла?
Аноним 25/10/19 Птн 12:53:31 52488331
>>524882
Это простая логика. Человек очень любит деньги, поэтому пойдет на все чтобы ими обладать, даже уверует в ЧД и других убедит, даже не заметит как произведет ограниченную выборку в эксперименте.
Аноним 25/10/19 Птн 12:55:36 52488432
>>524883
>в эксперименте
>астрономия

Адъ сошёл на раздел.
Аноним 25/10/19 Птн 13:00:21 52488533
>>524881
В центре Галактики вообще не ЧД, там другой объект, генерящий водород и антигравитацию. Конечно нагенеренный водород может скапливаться в крупные комки с повышенной гравитацией, но это по сути не чд, так как в них нет сингулярности.
Аноним 25/10/19 Птн 13:11:33 52488734
>>524881
На картинке вообще размытая хрень, словно 3.5 пикселя растянули и размыли. Конечно это не доказательство, а выдача желаемого за действительной. Вы еще в летающие тарелки уверуйте по 3.5 пиксельным НЛО на фотках.
Аноним 25/10/19 Птн 13:11:57 52488835
>>524885
>там другой объект, генерящий
Неужели там твоя мамаша
Аноним 25/10/19 Птн 13:15:19 52489036
>>524888
Там мамаша большинства звезд в галактике. Может быть и наше Солнце произведено из ее вещества.
25/10/19 Птн 13:20:27 52489137
>>524890
Виктор Амазаспович, вы?
Аноним 25/10/19 Птн 13:27:02 52489238
>>524891
Амбарцумян вроде как считает что звезды зарождаются сразу группами из некоего источника. Эти источники могут быть разбросаны по всей галактике. Возможно эти источники более мелкие варианты центрального галактического объекта.
Аноним 25/10/19 Птн 14:21:48 52489739
Можно ли понять квантовую механику интуитивно? Я не гум, но и не физик нихуя.
Аноним 25/10/19 Птн 14:46:33 52490240
>>524864
Этот чел >>524839 считает, что оно очевидно и фунамент там надежнее некуда. Я только гринтекстом выделил его посты, но сам так не считаю.
Аноним 25/10/19 Птн 14:48:25 52490341
>>524839
> Вот когда откроют квантовую теорию гравитации, тогда и поговорим.
Ну а хули ты тогда с такой уверенностью пиздишь про возможность сверхсветовых путешествий, если нихуя еще не открыли и не факт, что откроют вообще.
Аноним 25/10/19 Птн 14:49:32 52490442
>>524902
>Этот чел >>524839 считает, что оно очевидно

Это потому, что он верующий. Может он из секты червоточников, которые верят в червоточины.
Аноним 25/10/19 Птн 15:01:35 52490743
>>524903
Что такого невозможного в сверхсветовых путешествиях?
Аноним 25/10/19 Птн 16:27:59 52493444
>>524897
Ее и можно понять только интуитивно, проблема в переводе на общепонятный язык с языка математики. Даже с предыдущей теорией относительности (обеими) та же проблема.
25/10/19 Птн 16:29:06 52493645
>>524903
>не факт, что откроют вообще
Ты-то точно ничего не откроешь. Аргументы изложены, попытайся разобрать по частям и вникнуть в написанное.
>>524904
Мишань, спок.
Аноним 25/10/19 Птн 16:31:43 52493746
>>524884
Экспериментом является каждое астрономическое наблюдение, ну чего ты. Почитай вон на форуме астрономов, как первую межзвездную комету открывали, два месяца еще не прошло.
https://astronomy.ru/forum/index.php?topic=175643.80
Аноним 25/10/19 Птн 18:27:43 52496647
>>524937
>на форуме астрономов
Адъ становился гуще. Наблюдения становились эксперементами.
Люцифер начинал жаловаться на холод.
Аноним 25/10/19 Птн 18:29:10 52496948
>>524907
Нету способов это сделать. Законы физики запрещают.
Аноним 25/10/19 Птн 18:33:13 52497449
>>524936
> Аргументы изложены, попытайся разобрать по частям и вникнуть в написанное.
Твой аргумент - это:
Квантовую гравитацию должны открыть, отсюда следует, что червоточины абсолютно точно можно использовать для межзвездных перелетов.
То есть полный сумбур, а не аргумент.
Аноним 25/10/19 Птн 18:46:27 52498050
>>524969
Откуда ты уверен что законы физики это запрещают? Ты что знаешь эти законы?
Аноним 25/10/19 Птн 18:47:43 52498251
кот-шрёдингера-[...].jpeg (333Кб, 800x1145)
800x1145
>>524966
>Наблюдения становились эксперементами.
Как там, в 19-м веке, телеграф изобрели уже?
Аноним 25/10/19 Птн 18:50:58 52498652
>>524980
Да.
Если ты будешь двигаться быстрее скорости света, ты нарушишь принцип причинности. Но это в теории.
На практике на таких скоростях твой корабль распидорит обо всякие песчинки в межзвездном пространстве, а экипаж корабля будет жестко облучен.
Аноним 25/10/19 Птн 18:55:47 52498853
>>524986
>распидорит обо всякие песчинки в межзвездном пространстве
Это же в прямоточнике пофикшено. Все песчинки на пути жостко ионизируются и становятся топливом.
Аноним 25/10/19 Птн 18:57:01 52498954
Можно ли на космической станции уменьшать давление, увеличивая содержание кислорода для дыхания? И если да, то в каких пределах?
Аноним 25/10/19 Птн 19:00:08 52499255
Вещество набига[...].png (36Кб, 850x500)
850x500
>>524986
Ничего там не нарушается. Тупо разгоняешься и всё. А если кто говорит что НИЗЯЯЯ!!!!111 того можно смело игнорировать. Мало ли кто там что нафантазировал в своей теории.

>На практике на таких скоростях твой корабль распидорит обо всякие песчинки в межзвездном пространстве, а экипаж корабля будет жестко облучен.
В теории можно нарастить замедляющее песчинки поле вокруг корабля, как на пикче.
Аноним 25/10/19 Птн 19:01:17 52499356
В /sci/ толком ответа нормального не дали, так что спрошу тут:

Почему гравитация обратно пропорционально квадрату расстояния между телами? То есть я понимаю, что это всё можно вывести формулами, но хочется интуитивного объяснения. Например, я интуитивно понимаю, почему количество попадаемого света на тело подчиняется этому закону, но почему гравитация? Требуется такое объяснение, чтобы его можно было дать пиздюку 8-икласснику (не тупому), чтобы тот понял, почему формула такая, а не принял на веру.
Аноним 25/10/19 Птн 19:01:37 52499457
>>524989
Можно до 0.2 атм, было сделано, смысла особого не имеет, а пожароопасность резко повышается.
Аноним 25/10/19 Птн 19:05:10 52499658
>>524989
Да хоть до чистого кислорода можно при 0.2-0.3 атмосферах. Mercury, Gemini, Apollo так и летали, и за исключением сгоревшего Аполлона 1, проблем там не было.
Аноним 25/10/19 Птн 19:05:30 52499759
>>524993
https://ru.wikipedia.org/wiki/Закон_обратных_квадратов
Это тащемта не только для гравитации характерно, но и для любого излучения, и даже не только излучения, а, к примеру, для разлета осколков снаряда или даже для разлета содержимого взорвавшейся банки сгущенки.
Аноним 25/10/19 Птн 19:06:08 52499860
>>524986
Тело с массой нельзя разогнать до скорости света. При увеличении скорости масса растет по экспоненте, при приближении к скорости света масса стремится к бесконечности.
Поэтому со скоростью света летают только безмассовые частицы - фотоны, например.
Аноним 25/10/19 Птн 19:06:52 52499961
>>524993
Примерно то же самое что и у света. Гравитационный колодец в напряженности поля создает же центральное тело и воздействие этого тела распространяется во все стороны, как свет от Солнца.
Аноним 25/10/19 Птн 19:08:42 52500162
>>524997
Я в курсе закона. Но если в случае света увеличив площадь объекта, имеем больше частиц, втыкающихся в объект, то в случае гравитации увеличение площади (при той же массе) не увеличит гравитацию, то есть дело не во втыкании частиц. А в чем именно дело?
Аноним 25/10/19 Птн 19:08:52 52500263
>>524988
А как ты их всех ловить будешь? Не все же песчинки будут ровно по центру попадаться.
>>524992
А как ты будешь аналог Альбукерке создавать? Каким образом? Ладно, это неважно, на практике у тебя тупо не будет энергии для этого.
Аноним 25/10/19 Птн 19:11:57 52500564
>>525002
>А как ты будешь аналог Альбукерке создавать?
Я пока не знаю. Но в теории нет ничего необычного в этом.
Аноним 25/10/19 Птн 19:14:28 52500665
>>525001
У магнитного поля тоже частиц как бы нет, но тот же самый закон в его отношении тоже работает. Наверно можно себе как бы представить что есть некая интенсивность искривления пространства-времени, которая падает с расстоянием, но только чтобы примерно так почувствовать, не более.
Аноним 25/10/19 Птн 19:16:43 52500766
>>524998
>Тело с массой нельзя разогнать до скорости света.
Это манямир. Вон фотоны летают себе с цэ и массу имеют аш ню на цэ квадрат.
Аноним 25/10/19 Птн 19:17:10 52500967
Аноним 25/10/19 Птн 19:17:33 52501068
>>525006
>У магнитного поля тоже частиц как бы нет
Есть, его переносчик — фотон.
Аноним 25/10/19 Птн 19:18:44 52501169
>>525006
Ну я хочу примерно почувствовать, почему эта интенсивность падает квадратом, а не кубом
Аноним 25/10/19 Птн 19:19:47 52501270
>>525005
В теории ты что угодно можешь выдумать, хоть вечный двигатель. Но все это разобьется о суровую практику.
Для таких выдумок есть термин - фэнтези.
Аноним 25/10/19 Птн 19:21:10 52501371
>>525002
>Не все же песчинки будут ровно по центру попадаться.
Там магнитная воронка в тыщи километров, и не то чтобы постоянное магнитное поле. Прямоточный литак все песчинки по пути рвет в какаху, нагревая СВЧ. Там дури столько, что он и планету порвет, если чо.
Аноним 25/10/19 Птн 19:21:38 52501472
>>525011
У гравитаци на самом деле напряженность поля растет с удалением от тела. Гравитационная сила для пробного тела в поле гравитации направлена в сторону уменьшения напряженности поля.
Аноним 25/10/19 Птн 19:23:17 52501573
>>525007
Толсто. У фотона масса - 0.
Выдрал кусок формулы импульса фотона и доволен, идиот.
Аноним 25/10/19 Птн 19:23:39 52501674
>>525014
Но почему квадратом?
Аноним 25/10/19 Птн 19:23:59 52501775
>>525012
Да, на практике все сложнее, обязательно вылазят всякие детали, которые нужно учитывать.
Аноним 25/10/19 Птн 19:24:15 52501876
>>525013
И где ты энергии столько возьмешь? А магнитную воронку как сделаешь? Магнитное поле так то замкнутое.
Аноним 25/10/19 Птн 19:26:10 52501977
>>525011
Так тела взаимопритягиваются, то есть теряют одну степень свободы, по сравнению с одним телом, которое свободно фигачит фигню в пространство.
Аноним 25/10/19 Птн 19:27:39 52502078
>>525015
Фотон бьет о мишень с импульсом словно массивный шарик соответствующей массы. Это экспериментальный факт. А маняфантазии что масса фотона равна нулю никакими экспериментами не подкреплено. Что ты выберешь как физик изучающий реальность?
Аноним 25/10/19 Птн 19:29:26 52502279
>>525016
Примерно то же самое, но в обратную сторону. Представь что воображаемые гравитоны засасываются телом.
Аноним 25/10/19 Птн 19:29:59 52502380
>>525019
Не понял нихуя, почему теряют, что за тело, которое свободно фигачит фигню в пространстве?
Аноним 25/10/19 Птн 19:30:09 52502481
>>525020
> А маняфантазии что масса фотона равна нулю никакими экспериментами не подкреплено
> отличие массы фотона от нуля привело бы к дисперсии электромагнитных волн в вакууме, что размазало бы по небу наблюдаемые изображения галактик
Иди уроки делать, тролль мамкин.
Аноним 25/10/19 Птн 19:31:12 52502582
>>525018
Ну вот летит типа магнит, одним полюсом вперед. Плюс это поле модулируется для возбуждения встречных атомов. Оные ионизируются (в межзвездном газе много не надо, они и так там пухлые аж до сантиметра), фигачат в воронку, сжимаются и поскольку они в основном водород - термоядят, на выхлопе вся конструкция и летит.
Аноним 25/10/19 Птн 19:31:38 52502683
>>525022
Но ведь увеличение площади тела не увеличивает силу. Или ты имеешь в виду, что по всему объему тела засасываются? Тогда имеет смысл, спасибо.
Аноним 25/10/19 Птн 19:33:28 52502784
>>525023
Так куб ты откуда взял? Это именно одно неограниченное тело. При появлении другого тела связность растет на одно измерение, свобода на него же падает.
Аноним 25/10/19 Птн 19:33:44 52502885
>>525024
Там и так дисперсия. Например, радиоволны от вспышки сверхновой приходят позднее чем видимое излучение.
Аноним 25/10/19 Птн 19:34:28 52502986
>>525025
На такой скорости магнитное поле тебя не спасет. Песчинки тупо не будут успевать увлекаться им, т.к. не могут двигаться быстрее скорости света.
Плюс ты не ответил, откуда ты возьмешь энергию для магнитного поля в тыщи км.
Аноним 25/10/19 Птн 19:36:12 52503187
>>525028
Ты либо тролль, либо шизик-фанатик, подтасовывающий факты для поддержания своего манямирка.
Аноним 25/10/19 Птн 19:37:21 52503388
>>525026
>что по всему объему тела засасываются?
Как бэ да. Хотя на самом деле там скорее не гравитоны поглощаются, а часть их энергии и импульса поглощается. Но суть примерно такая же.
Аноним 25/10/19 Птн 19:40:58 52503989
>>525025
Сдует твое магнитное поле возле первой же звезды.
Аноним 25/10/19 Птн 19:41:27 52504090
>>525026
Нихрена ничего нигде никогда не засасывается, это ложная аналогия времен идеи "природа не терпит пустоты". Опровергнуто Торичелли с ртутным барометром. Сейчас считается, что все компрессоры работают на сжатие, и вакуумные насосы тоже.
Аноним 25/10/19 Птн 19:44:34 52504591
>>525029
>ты не ответил, откуда ты возьмешь энергию
>водород термояд
Ты не очень наблюдательный, верно?
Аноним 25/10/19 Птн 19:47:05 52504892
>>525040
Ты говоришь так словно вакуум это пустота.
Аноним 25/10/19 Птн 19:49:00 52505093
DSC0049.JPG (2706Кб, 5984x3376)
5984x3376
>>525039
Сейчас бы не отключать возле планеты обезьян.
Аноним 25/10/19 Птн 19:50:02 52505194
>>525045
А с каких это пор термояд стал панацеей? У него тоже есть свои ограничения.
Ты по сути хочешь в корабль сунуть магнитное поле соразмерное магнитному полю звезды. То есть по сути засунуть в корабль звезду.
Плюс ты задоджил довод, что песчинки не будут успевать расступаться.
Аноним 25/10/19 Птн 19:50:24 52505295
>>525050
А тормозить как будешь?
Аноним 25/10/19 Птн 19:51:40 52505396
>>525040
А что именно происходит тогда?
Аноним 25/10/19 Птн 19:53:33 52505597
>>525025
Давно выяснили что эта хуета будет тормозить сильнее чем давать импульс
Аноним 25/10/19 Птн 19:53:45 52505698
>>525033
Уф, как сформулировать тогда для пиздюка, чтобы не насыпать в голову каши, которая вызовет в будущем противоречия?
Аноним 25/10/19 Птн 19:53:50 52505899
>>525050
Отключишь, и тебя распидорит, лоооооооооооол!
Аноним 25/10/19 Птн 19:54:30 525059100
>>525055
А никак не построить её так даже теоретически? Прям твёрдо и чётко?
Аноним 25/10/19 Птн 19:57:14 525060101
>>525059
Построить можно, но ты никуда не улетишь так как сбор частиц будет тормозить сильнее чем последующий выброс этих частиц двигателем
Причем считали для термоядерных двигателей с миллионами импульса
Можно в принципе использовать для торможения по прилету
Аноним 25/10/19 Птн 20:01:54 525063102
>>525060
А какой тогда наиболее близкий действенный (пусть даже через дохулион лет) способ разогнаться?
Аноним 25/10/19 Птн 20:04:11 525065103
>>525063
взять с собой больше топлива и двигатель получше
Аноним 25/10/19 Птн 20:05:40 525067104
>>525056
Говори как есть. Каждый барион постоянно поглощает из вакуума энергию и импульс некой гравитонной среды, создавая вокруг себя в вакууме область с градиентом энергии и импульса этой среды, поэтому попавшее в этот градиент частицы испытывают нескомпенсированное давление этой среды с внешней стороны. Если не понимает, то расскажи ему за первую теорию гравитации от Лесажа.
Аноним 25/10/19 Птн 20:07:12 525069105
>>525065
На реактивном движении не получится, с этим надо просто смириться. Это надо будет брать с собой топлива массой с планету или около того.
>>525063
Разогнаться мало, надо еще как то затормозить.
Аноним 25/10/19 Птн 20:16:39 525074106
>>525069
Хокенк предлагал же лазерами ебошить. Почему так нельзя?
Аноним 25/10/19 Птн 20:18:26 525075107
>>525067
Ладно, спасибо анонче
Аноним 25/10/19 Птн 20:19:09 525076108
>>525074
Лазеры тупо сожгут дрон.
Аноним 25/10/19 Птн 20:19:53 525078109
>>525069
Даже с уи 100000000000?
>>525074
Можно, но с расстоянием увеличивается размер пятна, а для короткого разгона будет слишком большое ускорение
Аноним 25/10/19 Птн 20:20:36 525079110
Скажите, если космонавт спрыгнет с мкс в сторону Земли (оттолкнется ногами от обшивки), то он будет падать по спирали или не будет падать, а продолжит крутиться на орбите?
Аноним 25/10/19 Птн 20:21:09 525081111
>>525078
> уи 100000000000
Чё эта?
Аноним 25/10/19 Птн 20:24:00 525084112
>>525081
удельный импульс
грубо говоря эффективность двигателя
Аноним 25/10/19 Птн 20:25:59 525086113
>>525079
Продолжит крутиться по орбите, чуть отличной от орбиты МКС.
Аноним 25/10/19 Птн 20:26:15 525087114
Аноним 25/10/19 Птн 20:26:24 525088115
>>525078
>но с расстоянием увеличивается размер пятна
Сделать многоступенчатый лазер. То есть запускаем корабль вместе с лазером, разгоняем с Земли пока можем, а потом отстыковываемся и ебошим лазером уже из космоса. Или хуйню сказал?
Аноним 25/10/19 Птн 20:26:31 525089116
>>525084
Ну где ты такой возьмешь? Даже если аннигилировать материю будешь, все равно этого недостаточно. Да и межзвездные расстояния просто адовые. Даже фотонам лететь годами.
Аноним 25/10/19 Птн 20:27:04 525090117
>>525075
Если совсем просто говорить за природу гравитации, то это аналог температуры в газе. Если сильно охладить два шарика и подержать их друг возле друга в нагретом воздухе, то они будут приталкиваться более горячим воздухом.
Аноним 25/10/19 Птн 20:27:54 525091118
>>525086
А почему? Он же по идее получил ускорение в сторону Земли?
Аноним 25/10/19 Птн 20:29:36 525092119
>>525090
Ну такие аналогии слишком далёки, не хочется прям совсем научпопить
Аноним 25/10/19 Птн 20:30:02 525093120
>>525091
Однократное. И перешел на другую орбиту. Вот если бы у него был двигатель, которым бы он постоянно разгонялся, то тогда бы упал.
Аноним 25/10/19 Птн 20:31:08 525094121
>>525093
Что значит однократное? Оно куда то пропадет?
Аноним 25/10/19 Птн 20:31:19 525095122
>>525089
ну, я нули на отъебись ставил, однако некоторые варианты термоядерных двигателей позволяют достичь много миллионов импульса
даже на более технологически возможной пылевой плазме можно сделать миллион
Аноним 25/10/19 Птн 20:32:41 525097123
>>525094
оно не возникает с момента толчка от мкс
Аноним 25/10/19 Птн 20:33:11 525098124
>>525094
Нет, бля. Если ты во вторник подпрыгнул, то будешь теперь до конца недели подлетать.
Аноним 25/10/19 Птн 20:33:11 525099125
>>525095
Ну это же все равно топливо надо на своем горбу везти. А чем больше топлива, тем тяжелее корабль, тем труднее его разогнать. Реактивные движки ппц какие неэффективные.
25/10/19 Птн 20:34:13 525101126
>>524974
Это не мой аргумент. Про "абсолютно точно" я нигде не говорил. Проваливай.
>сумбур
у тебя в голове. Ты долбился в глаза и проглядел абсолютно все мои аргументы, касающиеся сути дела.
Аноним 25/10/19 Птн 20:34:24 525102127
>>525097
Подожди, а что гасит ускорение, если мы в вакууме?
Аноним 25/10/19 Птн 20:34:38 525103128
>>525099
Даже если ёбнуть фотонный двигатель?
Аноним 25/10/19 Птн 20:36:47 525105129
Почему на ядерном импульсном двигле не полететь? Проект орион разгонялся до 10к км/сек. Вроде не так критично при столкновении. Лететь 130 лет, но почему не запилить корабль поколений?
Аноним 25/10/19 Птн 20:40:01 525106130
>>525102
ты зачем путаешь скорость и ускорение?
ускорение это изменение скорости
вот бы стоишь на мкс, у тебя скорость как у станции и орбита такая же
оттнолкнулся ногами - придал ускорение - изменил скорость и орбиту
а дальше все
со временем упадешь на землю так как на высоте мкс есть немного атмосферы и она тормозит
>>525099
нет, такой импульс это очень эффективно
по солнечной системе можно на похуях летать за пару дней куда угодно
>>525103
для него совершенно невменяемые энергии нужны, а энергия это опять масса топлива для реактора и сам реактор
>>525105
нахуй не нужно
Аноним 25/10/19 Птн 20:40:08 525107131
>>525105
при столкновении с межзвездной пылью*
быстрофикс
Аноним 25/10/19 Птн 20:40:11 525108132
>>525101
Твои аргументы - это кабы было то, кабы было это, так все было бы заебись. Типа существует такая то теория и она не запрещает червоточины, а раз не запрещает, то значит червоточины реально есть. Хотя все эти теории еще не доказаны.
По факту ты ничего толкового так и не сказал. Не объяснил, почему должны существовать червоточины, не объяснил каким образом их создавать. Потому что не разбираешься в теме и уповаешь на то, что кто нибудь когда нибудь все это придумает.
Аноним 25/10/19 Птн 20:40:49 525109133
>>525103
А чем фотонный двигатель лучше других?
25/10/19 Птн 20:41:07 525110134
>>525012
>В теории ты что угодно можешь выдумать, хоть вечный двигатель
Нет, вечный двигатель как раз-таки противоречит теории, так как именно в теории он нарушает, скажем, примерно так всё, от закона сохранения энергии до унитарности в квантовой механике. Здесь гораздо ближе и точнее аналогия с трудностями создания квантового компьютера.
Аноним 25/10/19 Птн 20:42:03 525111135
>>525109
Рабочее тело не нужно
Аноним 25/10/19 Птн 20:42:46 525113136
>>525106
>для него совершенно невменяемые энергии нужны
Ну уменьшенная версия, для которой был сделан расчет, весит 100к тонн. Это примерно 100 заправленных Старшипов по массе. Не такие гигантские затраты для такого проекта.
Аноним 25/10/19 Птн 20:43:08 525114137
Кстати правильно ли я понимаю что фотонный двигатель с более-менее тягой будет испускать луч смерти?
Аноним 25/10/19 Птн 20:43:17 525115138
>>525109
Удельный импульс в 299кк
Аноним 25/10/19 Птн 20:43:50 525117139
>>525113
покажи проект
кстати ты массу реактора, топлива и радиаторов забыл
Аноним 25/10/19 Птн 20:44:12 525118140
>>525110
Создание своих червоточин для межзвездного перемещения тоже противоречит теориям и здравому смыслу
Аноним 25/10/19 Птн 20:44:39 525119141
>>525115
А он не сожжет корабль?
25/10/19 Птн 20:45:39 525120142
>>525108
>Не объяснил, почему должны существовать червоточины
В глаза долбишься, толстяк:
>>524754
>Червоточины - вообще один из самых очевидных квантовогравитационных эффектов. Тоталитарный принцип Гелл-Манна, опять же: квантовогравитационный аналог фейнмановского интеграла по траекториям должен, помимо метрик с тривиальной топологией, включать также и метрики с топологией нетривиальной - точно так же, как при расчете вероятностей взаимодействий частиц на коллайдерах необходимо учитывать "петли" виртуальных частиц. Почему? Да потому что блять нет ни одного фундаментального квантовомеханического принципа, ни одной фундаментальной симметрии, которая бы их запрещала.
Если ты неспособен понять, что под "нетривиальной топологией" имелись в виду именно червоточины, то о чем вообще с тобой говорить?
Аноним 25/10/19 Птн 20:45:47 525121143
>>525119
вообще должен, прожекторы же пиздец греются
Аноним 25/10/19 Птн 20:47:18 525124144
>>525120
Чел, тут как бэ есть одно но. Квантовой гравитации нет.
25/10/19 Птн 20:47:32 525125145
>>525118
>тоже противоречит теориям
Каким?
>здравому смыслу
Здравый смысл в физике давно ушел нахуй, по очевидным причинам.
Аноним 25/10/19 Птн 20:49:15 525127146
>>525119
Ну мы говорим о сверхчеликах через 500-1000 лет. Наверное научатся пиздец эффективно отводить тепло
Аноним 25/10/19 Птн 20:49:44 525128147
Аноним 25/10/19 Птн 20:49:46 525129148
>>525125
Ну скажи, как ты будешь открывать червоточину? Допустим, есть Солнце и есть Альфа Центавра. Тебе надо открыть червоточину ведущую в окрестности второй.
Эсли все это так замечательно описано теориями, то будь добро, опиши, как это будет происходить?
Аноним 25/10/19 Птн 20:50:23 525130149
>>525127
Угу. Прямиком в варп.
25/10/19 Птн 20:52:39 525134150
>>525124
>Квантовой гравитации нет
Если теория еще не построена, еще не значит, что ее в принципе нельзя построить. А строить ее нужно, так как иначе остаются необъясненными термодинамика черных дыр и начальные условия для космологической инфляции.
>>525129
Харэ толстить. Конкретный способ открытия червоточины можно будет знать лишь после построения теории. Сам факт их принципиальной возможности можно утверждать уже сейчас. Так понятно?
Аноним 25/10/19 Птн 20:57:31 525137151
>>525134
> Сам факт их принципиальной возможности можно утверждать уже сейчас.
Нет, нельзя. Т.к. теория еще не построена. Как бы тебе не хотелось. Иначе это называется предвзятость.
А может квантовая гравитация неверна, откуда ты знаешь? Ты же не доказал еще, что это работает.
Аноним 25/10/19 Птн 21:00:07 525139152
>>525134
Чел, сам факт существования червоточин не будет означать, что люди смогут их создавать.
Вон звезды абсолютно точно на 100% существуют. И что? Люди могут создать свою звезду? То то же.
Аноним 25/10/19 Птн 21:06:57 525144153
.jpg (51Кб, 552x552)
552x552
>>525139
>Люди могут создать свою звезду?
Да могут.
Аноним 25/10/19 Птн 21:11:39 525145154
>>525115
Люди не переживут такие перегрузки, лол.
Аноним 25/10/19 Птн 21:14:25 525148155
>>525145
>удельный импульс
>перегрузки
ты дурак?
25/10/19 Птн 21:16:35 525149156
>>525137
>Нет, нельзя. Т.к. теория еще не построена. Как бы тебе не хотелось. Иначе это называется предвзятость.
Хуле ты мне втираешь за методологию? Общие принципы квантовой механики одинаковы в любой квантовой теории. Применение их к гравитации приводит к неизбежности червоточин независимо от деталей теории, точно так же, как виртуальные "частицы" существуют в любой квантовой теории поля.
>квантовая гравитация неверна
Те проблемы современной теоретической физики:
>термодинамика черных дыр и начальные условия для космологической инфляции
требуют решения, и как минимум в случае с термодинамикой нам известно, что в известных нам случаях (тепловое излучение АЧТ, значение теплоемкостей etc.) решение было именно в построении квантовой теории. Так что то или иное квантование гравитации как минимум ожидаемо. Не говоря уже о чисто теоретических аргументах типа того, что по крайней мере для некоторых очень важных квантовогравитационных моделей можно найти эквивалентную формулировку в виде квантовополевой теории без гравитации, и относительно внутренней самосогласованности этой теорий у теоретиков сомнений нет.
>>525139
С другой стороны, в природе "нет" многих вещей, которые способны создать люди. Так что этот твой аргумент инвалид.
Аноним 25/10/19 Птн 22:36:27 525171157
>>525149
> Хуле ты мне втираешь за методологию? Общие принципы квантовой механики одинаковы в любой квантовой теории. Применение их к гравитации приводит к неизбежности червоточин независимо от деталей теории, точно так же, как виртуальные "частицы" существуют в любой квантовой теории поля.
Ну раз так, то хуле ты не можешь описать, как создавать червоточины? Общие принципы квантовой механики же одинаковы в любой квантовой теории. Камон. Опиши тогда создание червоточины с помощью квантовой механики. Все же уже готово.
> в природе "нет" многих вещей, которые способны создать люди.
Да, но по соотношению энергии, необходимой на создание, вещи, созданные людьми никак не сравнятся с астрономическими объектами наподобие звезд, черных дыр и червоточин.
Аноним 26/10/19 Суб 18:27:01 525257158
26/10/19 Суб 18:36:10 525259159
>>525171
>хуле ты не можешь описать, как создавать червоточины
Грубо говоря, для этого нужно знать, как именно выглядит аналог гамильтониана/лагранжиана для квантовой теории гравитации. Он пока не известен.
>Все же уже готово
Не передергивай, маня.
>никак не сравнятся с астрономическими объектами
Никто не утверждает, что радиус горловины червоточины должен быть "астрономическим". Опять же, точные значения "энергии", нужной для создания червоточин, можно будет оценить лишь когда станет известен гамильтониан/лагранжиан для квантовой теории гравитации.
Аноним 26/10/19 Суб 19:07:58 525263160
>>525259
Ты лучше скажи, как ты направление червоточины будешь задавать на практике? Бумажки исписывать - это одно, а практика - совсем другое. Интуиция мне подсказывает, что для этого открывающие червоточину должны находиться на обоих ее концах. А для этого нужно сначала добраться до места назначения.
Аноним 26/10/19 Суб 19:10:16 525264161
>>525259
> Опять же, точные значения "энергии", нужной для создания червоточи
Ты собираешься пространство искривлять, причем не пару кубометров, а пространство порядка световых годов. Тут не надо быть нобелевским лауреатом, чтобы понять, что для этого понадобится астрономическое количество энергии.
26/10/19 Суб 19:14:07 525265162
>>525263
>Интуиция мне подсказывает
Квантовая физика - дохуя контринтуитивная хрень, школотун.
>>525264
Ты ебанулся? Смысл интересной с практической точки зрения червоточины как раз в том, что она короче "световых годов". Иначе нахуя она такая длинная нужна, если по ней лететь как до альфы Центавра?
Аноним 26/10/19 Суб 19:22:40 525266163
>>525265
> Смысл интересной с практической точки зрения червоточины как раз в том, что она короче "световых годов".
Смысл в том, что ты искривляешь пространство, как в научпопе же сгибают лист бумаги, да, чтобы показать суть кротовой норы, и потом открываешь червоточину, чтобы сократить путь.
Аноним 26/10/19 Суб 19:25:38 525267164
>>525266
Нет, "большое" пространство при этом никуда не сгибается. Лист бумаги сгибают потому, что просто иначе на практике это не покажешь.
Аноним 26/10/19 Суб 19:27:16 525268165
>>525267
Нет, сгибается. Иначе как ты короткий путь построишь? Либо искать область, где оно уже итак согнуто, либо самому сгибать
Аноним 26/10/19 Суб 19:39:00 525273166
>>525268
Нет, не сгибается. В том-то и дело! Из-за неевклидовой метрики в червоточине она оказывается короче, чем путь в объемлющем (плоском) пространстве, хотя объемлющее пространство осталось плоским.
Аноним 26/10/19 Суб 19:45:15 525276167
>>525273
Как ты себе это представляешь? Вот ты говоришь, что пространство плоское. Представь себе лист бумаги. Как ты построишь путь между двумя точками короче прямой, если он плоский? Никак. Только если согнуть его. То есть плоское евклидово пространство должно стать неевклидовым.
Аноним 26/10/19 Суб 20:07:41 525279168
>>525276
>Как ты построишь путь между двумя точками короче прямой, если он плоский? Никак.
Если метрика в кротовой норе неевклидова, то расстояние между точками там может быть как больше, так и меньше, чем на плоскости (в зависимости от конкретной метрики).
Аноним 26/10/19 Суб 20:10:24 525282169
>>525279
Там это где? В волшебной стране?
Аноним 26/10/19 Суб 20:27:52 525289170
>>525282
В анусе твоей жирной мамаши-щлюхи, а ты как думал? головине кротовой норы.
Аноним 26/10/19 Суб 20:35:59 525291171
>>525289
А червоточина что, по твоему, где то в другом пространстве находится?
Аноним 26/10/19 Суб 21:00:24 525300172
>>525291
Червоточина "не находится" в пространстве, она и есть кусок пространства. Просто "свойства" пространства-времени в разных его частях могут быть разными.
Аноним 26/10/19 Суб 21:11:04 525305173
>>525300
Ну и как ты будешь менять свойства пространства так, чтобы во-первых оно вело в точку за десятки-сотни световых лет от тебя, и, во-вторых, становилось неевклидовым настолько, чтобы этот путь сократить? Где ты будешь столько энергии брать, чтобы совершать такое колдунство над пространством? А ты уверен вообще, что свойства пространства на практике возможно изменять человеческими силами?
Аноним 26/10/19 Суб 23:25:59 525322174
Представим, что мы привезли на луну огромное количество сверхлёгкого песка и заполнили им кратер до краев. Можно ли использовать такой песок наподобие водяной подушки для посадки в него кораблей литобрекингом со скоростями около 70 метров в секунду?
Аноним 26/10/19 Суб 23:46:29 525326175
stage03.gif (682Кб, 76x375)
76x375
>>524797 (OP)
Сколько может забросить центральный блок Энергии без боковушек на НОО?
27/10/19 Вск 00:16:28 525330176
Аноним 27/10/19 Вск 01:07:23 525338177
>>525326
вроде тяги не хватит для взлёта
Аноним 27/10/19 Вск 01:52:07 525341178
>>525326
Ноль. Его не существует.
Аноним 27/10/19 Вск 02:51:46 525342179
>>524993
наглядно. но не физично.

представь что тебе в руку стреляют три раза из автомата в упор. это три силовых воздействия. сейчас они у нас будут "линейной" зависимости от расстояния.

автомат отдаляется от тебя на 100 метров, и все также стреляет в тебя три раза. ты опять ловишь три воздействия. линейная зависимость

с ростом расстояния воздействие будет все слабее. и на расстоянии 3км. если конечно снайпер попадет эти три выстрела уже тебя не пробьют, а только долетят.линейная зависимость.

квадратичная зависимость.
представь что граната ф-1 ебнет у тебя в руке. половина гравитационных волн осколков - проделает путь через тебя.
теперь граната взрывается в 10 метрах от тебя. но на твой сектор направлена уже не половина осколков, а меньше.
граната взрывается в 50 метрах от тебя. ни один осколок не попал в тсовй сектор. это квадратичная зависимость.

почему зависимость квадратичная а не кубическая... Потому что либо мир на самом деле 2д с псевдопроекцией. учеюники скрывают.
Аноним 27/10/19 Вск 02:59:27 525343180
A3776AA8-B7AC-4[...].jpeg (986Кб, 1951x1125)
1951x1125
Посмотрел этот видос:
https://youtube.com/watch?v=gcvq1DAM-DE
Но вот этот момент не до конца понял. Если верить графику, то разные части тела в один и тот же момент, должны быть в разных местах во времени. То есть в реальной жизни ведь объект при падении на таком графике будет перпендикулярен земле. Короче нихуя не понял, анон, поясняй.
Аноним 27/10/19 Вск 07:30:15 525345181
Какие профиты можно получить от освоения других планет нашей системы? Зачем на Луну рвутся, более-менее понятно - там можно устроить лабораторию и опорный пункт для дальнейшего освоения космоса, осуществлять добычу топлива, размещать вредные производства, складировать ядрёное говно.
Аноним 27/10/19 Вск 09:29:18 525356182
>>525330
Короче, на мои вопросы ты не способен ответить конкретикой. Если подуести итоги, то у твоей теории слишком много невыполненных условий, чтобы считать ее достоверной:
1) нужно, чтобы существовала квантовая гравитация;
2) нужно знать, как создавать эти червоточины;
3) нужно знать, как задавать направление этой червоточины;
4) нужно знать, как менять свойства пространства-времени;
5) нужно знать как менять метрику пространства-времени;
6) нужно знать, как сделать этот тоннель стабильным во времени;
7) неизвестно, сколько энергии понадобится на эти действия, но судя по тому, что пространство вообще способно искривляться только возле очень массивных объектов типа черных дыр, то это астрономические порядки энергии.
Ни для одной из этих проблем ты не смог найти внятного решения. Все твои ответы можно обобщить, как: "Вот построят теорию квантовой грпвитации, тогда заживем!"
Вердикт: очередное фентези, автор которого - верун-фанатик.
Аноним 27/10/19 Вск 09:33:48 525357183
Двач, что такое Черная дыра? Простыми словами, джля аутиста.
Аноним 27/10/19 Вск 09:44:31 525359184
>>525357
На второй ОП пик посмотри.
Аноним 27/10/19 Вск 09:49:46 525360185
>>525359
Так, значит горизонт событий - это такой себе клапан, который всех впускает и никого не выпускает.
Сингулярность - это состояние пространства-времени, которое является конечной точкой путешествия объектов в черной дыре.
Но как это работает? Откуда берется такая мощная гравитация?
Аноним 27/10/19 Вск 10:30:32 525364186
>>525338
Бля. А если наполовину заправить, себя хоть выведет?
Аноним 27/10/19 Вск 12:03:41 525369187
>>525360
Гравитация ниоткуда не берется, она всегда на месте была. Не забывай, что хотя сингулярность и точка, а сама черная дыра (не СМЧД) всего несколько десятков километров в диаметре, туда упаковано колоссальное количество вещества, несколько солнечных масс даже в самых маленьких из обнаруженных черных дыр.

Чтобы противостоять такому мощному тяготению, нужны не менее мощные отталкивающие силы, и когда в выгорающей массивной звезде они ослабевают, черные дыры и появляются.
Аноним 27/10/19 Вск 15:00:36 525391188
Аноним 27/10/19 Вск 15:38:31 525400189
>>525357
Эта такая теоретическая хуита. Знаишь, вот сначала берут несколько начальных положений и от них потом отталкиваются. Если делают охуитительную теорию, то некоторые начальные положения берут тупо от балды, надеясь на авось или хрен знает на что надеясь, типа яскозал что вот так оно и ниипет. Еще часто так получается что всего не упомнишь или боишься сложности или еще чего и начальные положения получаются тупа недостаточны, в этом случае точность теории падает и границы применимости уменьшаются. Потом начинают мозговать какие следствия из этих положений вытекают. Естественно, если у теории проблемы с аксиоматикой, то будут вылазить всякие парадоксы и объекты с охуительными свойствами. Вот пример таких объектов как раз и есть чоние дирки.
Аноним 27/10/19 Вск 18:52:21 525452190
latest.jpg (94Кб, 960x900)
960x900
Ебать тут срач однако за достижение скорости света и быстрее. Однако, смею напомнить, что даже стартрековый варп-двигатель это будет ПИЗДЕЦ медленно
https://www.youtube.com/watch?v=G-oI5Y4sAi8

Да, конечно, это будет гораздо быстрее чем сейчас, но о галактике можно просто нахуй решительно забыть. Червоточины значительно лучше. Вся надежда на них
Аноним 27/10/19 Вск 21:17:59 525469191
>>525356
Вердикты своей мамке будешь выносить, Мишань. Тебе русским языком сказали: как только квантовая теория гравитации будет создана (а ее не может не быть, так как классическая теория гравитации - ОТО - неполна, и опыт разрешения аналогичных парадоксов в физике подсказывает, что гравитация должна быть проквантована), станут ясны и ответы на твои вопросы. Если ты знаешь, как работает теоретическая физика, это должно быть очевидно. Но ты этого не знаешь, и потому что-то там пиздишь за верунов. Напомню, что именно квантовые теории являются наиболее точно подтвеждаемыми экспериментально теориями, поэтому именно на их основании можно делать "наиболее уверенные" высказывания о чем-бы то ни было в науке. Просто иди нахуй.
Аноним 27/10/19 Вск 21:22:35 525470192
>>525452
Это че, на самом топовом стар трековом варпе до млечного пути 50 лет пердолить?
Аноним 27/10/19 Вск 21:22:56 525471193
>>525452
Кстати, чаю тебе за старгейт
Аноним 27/10/19 Вск 21:23:22 525473194
>>525343
Анон, ты ответишь мне?
Аноним 27/10/19 Вск 21:23:26 525474195
>>525469
>квантовые теории являются наиболее точно подтвеждаемыми экспериментально теориями
Толстовато.
Аноним 27/10/19 Вск 21:23:54 525475196
Аноним 27/10/19 Вск 21:25:28 525478197
Аноним 27/10/19 Вск 21:38:48 525482198
Анон, есть ли какой сайфай про совсем далёкую эпоху, когда во вселенной почти всё перетухло и люди (или какое ебическое нечто вместо людей будет) пытаются выживать рядом с белыми карликами?
Аноним 27/10/19 Вск 23:17:20 525505199
>>525474
Чел, он не тролль, он и вправду так считает. Я же говорю, фанатик.
Есть веруны во всемогущего бога, а есть веруны во всемогущую квантовую механику. Это один из них.
Аноним 27/10/19 Вск 23:27:55 525507200
>>525469
Даже если построят теорию квантовой гравитации, то отсюда все равно не следует, что люди смогут создавать червоточины. У тебя ошибка в мышлении.
Вот тебе пример. Открыли люди ядерные взаимодействия. Описали, как из одних элементов путем добавления нужного количества протонов и нейтронов получить другие элементы И что? У нас есть теперь легко доступная трансмутация по твоей логике? Медь в золото превращает кто нибудь? Нет. Потому что на это нужно неимоверное количество энергии. И проще копаться в земле, выискивая крупицы золота, чем делать все по этим бумажным теориям.
Так вот, представь, на трансмутацию нужно неимоверно много энергии, но по сравнению с открытием червоточин это просто пшик. Потому что ткань пространства могут искривлять только очень массивные астрономические объекты.
Аноним 28/10/19 Пнд 00:08:42 525512201
>>525507
>Вот тебе пример. Открыли люди ядерные взаимодействия. Описали, как из одних элементов путем добавления нужного количества протонов и нейтронов получить другие элементы И что? У нас есть теперь легко доступная трансмутация по твоей логике? Медь в золото превращает кто нибудь? Нет. Потому что на это нужно неимоверное количество энергии. И проще копаться в земле, выискивая крупицы золота, чем делать все по этим бумажным теориям.
Это вопрос не количества энергии, а качества её фокусировки.

Сейчас мы всё что угодно можем делать на микрометровом масштабе и с большим геморроем на нанометровом. А тонкие манипуляции с ядрами это пикометры и фемтометры. Здесь мы пока можем только грубо дубасить молотком в надежде, что сколько-то процентов правильно прореагирует.

Но никаких принципиальных ограничений на тонкость манипуляций у нас нет. Рано или поздно появятся всякие аттосекундные гамма-лазеры и прочая поебень. Возьми дистанцию между химией 17 века и чудесами фотолитографии конца 20 века, и примерно отмерь то же самое в будущее.
Аноним 28/10/19 Пнд 00:24:10 525515202
Аноним 28/10/19 Пнд 00:25:30 525516203
>>525512
>Возьми дистанцию между химией 17 века и чудесами фотолитографии конца 20 века, и примерно отмерь то же самое в будущее.
Но мы ведь уже наверху S-образной кривой развития.
28/10/19 Пнд 00:37:30 525518204
Аноним 28/10/19 Пнд 00:50:57 525523205
>>525512
Не пытайся ему что-то доказать. Если дебич не одупляет разницу между религией и методами науки, то что ему вообще можно рационально доказать? Он из своего куцего межушного ганглия достал идею, что на создание кротовых нор почему-то нужно ДОХУЯ энергии, и теперь носится с этим высером по треду, как угорелый, хотя обосновать этот тезис он не может ничем, кроме как апелляцией к якобы огромным масштабам задачи.
Аноним 28/10/19 Пнд 01:07:48 525527206
>>525516
>Но мы ведь уже наверху S-образной кривой развития

Это лишь гипотеза, что якобы мы где-то там наверху и прогресс затормозился. Есть еще гипотеза, что наоборот ускорился. И та и другая это лишь гипотезы, ведь никому не дано знать будущего.
Аноним 28/10/19 Пнд 01:10:55 525528207
>>525523
>Он из своего куцего межушного ганглия достал идею, что на создание кротовых нор почему-то нужно ДОХУЯ энергии,

А почему бы и нет? Например, Луис Крейн и Шон Вестморленд, предложили гипотезу что на создание одной чёрной дыры весом в миллион тонн, радиус которой будет меньше радиуса атома, придётся потратить около года. Для этого необходимо будет построить гигантскую солнечную панель и создать мощный гамма-лазер на сотню гигаватт, который сгенерирует сгусток излучения, способный в дальнейшем коллапсировать с образованием чёрной дыры.
Дыра немедленно взорвется и возможно либо создаст кротовую дыру в другую вселенную, либо нет.
Аноним 28/10/19 Пнд 02:29:21 525530208
>>525516
>Но мы ведь уже наверху S-образной кривой развития.
Иерофант в треде?
Этих кривых развития уже было дохуя, в том числе с откатами назад. Но общий исторический тренд строго вверх.

>>525523>>525528
Эти вопросы вообще касаются не фундаментальной науки, а прикладной. Фактор "ДОХУЯ/НЕ ДОХУЯ" - это исключительно производная от фокусировки, перераспределения и кпд.

Вот сколько энергии нужно, чтобы вывести тонну на орбиту? Очевидно, ДОХУЯ. А на самом деле нужно НИХУЯ, потому что если у нас огромный постоянный грузопоток между землёй и орбитой, то одни грузы мы тормозим, другие ускоряем, и в среднем задача сводится к перераспределению энергии от одних грузов к другим с минимизацией потерь об атмосферу. Т.е. энергозатраты на строительство в космосе гигантских городов и на добычу в космосе гигантских ресурсов на самом деле примерно нулевые с идеалистической точки зрения. Задача эта чисто инженерная. Очень сложная и громоздкая, но инженерная. С точки зрения фундаментальной науки здесь вообще никаких проблем нет.

Что там будет с червоточинами - пока даже гадать рано. Есть общий принцип, что если где-то убыло - где-то прибыло. Есть общий принцип, что замкнутый хорошо заизолированный процесс будет вечно идти по инерции. Даже с демоном Максвелла всё не так однозначно, а с энтропией во всяких экзотических моделях типа диска Эренфеста и подавно.

Я бы сформулировал так: при инженерно-идеальном исполнении энергозатраты на любой обратимый процесс равны нулю. Насколько червоточины в эту концепцию вписываются - решать инженерам, которые в этой индустрии сколько-то столетий поработают.
Аноним 28/10/19 Пнд 02:31:45 525531209
Аноны, мысленный эксперимент:
Что будет с миром, если "отключить" по очереди по одному взаимодействию, оставляя 3 других?

1. Сильное
2. Слабое
3. Электромагнитное
4. Гравитационное

?
Аноним 28/10/19 Пнд 03:15:53 525532210
Аноним 28/10/19 Пнд 03:34:36 525533211
>>525530
>Но общий исторический тренд строго вверх
Никто не спорит, другое дело, что новые достижения будут всё реже и реже происходить и стремиться к некой горизонтали, которая явно ниже ёбанных червоточин.
Аноним 28/10/19 Пнд 03:46:18 525534212
>>525530
>при инженерно-идеальном исполнении энергозатраты на любой обратимый процесс равны нулю.
К сожалению в природе обратимых процессов не бывает. И не энергозатраты могут быть равны нудю, а разница между той энергией что затрачена на некий процесс и той энергией что этот процесс произвел. Вот, к примеру, возьмем ракету Протон, какую работу она совершит выводя максимальную полезную нагрузку на НОО. А теперь сравни это число с суммарными затратами энергии на изготовление этой ракеты, на заправку.
Аноним 28/10/19 Пнд 03:51:03 525535213
>>525531
>1. Сильное
Нуклоны моментально развалятся и мир каким мы его знаем исчезнет.
>Слабое
Звезды немедленно погаснут, потому что термоядерные процессы станут невозможны.
>Электромагнитное
Атомы и молекулы распадутся.
>Гравитационное
Макроскопические тела развалятся, орбиты распадутся, планеты и звезды рассыпятся на мелкие частицы.
Аноним 28/10/19 Пнд 03:52:35 525536214
>>525533
>другое дело, что новые достижения будут всё реже и реже
Никому не дано знать будущего. Если вы видите высказывания подобного рода - знайте, что к науке они не имеют никакого отношения.
Аноним 28/10/19 Пнд 05:05:30 525541215
>>525536
Предсказания технического прогресса на 10 лет вперёд как правило сильно отличаются от реальности, а тут аноны на квинтиллионы лет нагадали.
Аноним 28/10/19 Пнд 05:37:40 525542216
>>524797 (OP)
Если сделать солнечный парус площадью 1 квадратный километр, прицепить к нему сычевальню и запустить с орбиты Земли, то за сколько он долетит до Проксима Кентавра b?
Аноним 28/10/19 Пнд 05:43:28 525543217
>>525533
>Никто не спорит, другое дело, что новые достижения будут всё реже и реже происходить и стремиться к некой горизонтали, которая явно ниже ёбанных червоточин.
Курс доллара, евро, биткоина и йены на декабрь 2020 года, пожалуйста. И анус с распиской. Тогда и поговорим. В противном случае пруф на методологию своей хуйни.

>>525534
>К сожалению в природе обратимых процессов не бывает.
Разогнал электровоз, покатился по инерции, остановил электровоз, рекуперировав энергию.
>Вот, к примеру, возьмем ракету Протон, какую работу она совершит выводя максимальную полезную нагрузку на НОО. А теперь сравни это число с суммарными затратами энергии на изготовление этой ракеты, на заправку.
А теперь берём космический лифт и поднимаем на ГСО тонну говна. Ловим там же другую тонну говна и опускаем на землю, рекуперируя энергию.
Аноним 28/10/19 Пнд 05:49:30 525544218
>возьмем ракету Протон
Зачем ты берёшь это древнее одноразовое говно?
Аноним 28/10/19 Пнд 05:52:30 525545219
>>525543
>Разогнал электровоз, покатился по инерции, остановил электровоз, рекуперировав энергию.

Но КПД процесса рекуперации по законам физики не может быть 100 процентов да и кинетическая .
Энергия необратимо рассеялась в виде тепла т.е. твое желание чтобы "энергозатраты на любой обратимый процесс равны нулю." невыполнимо.
Аноним 28/10/19 Пнд 05:53:09 525546220
>>525544
Возьми любую другую ракету которая тебе нравится и рассчитай.
Аноним 28/10/19 Пнд 07:34:08 525556221
>>525545
>Но КПД процесса рекуперации по законам физики не может быть 100 процентов
Это с каких таких хуёв? Возьми маятник в вакууме на магнитном подвесе, и будет он у тебя дрочиться до скончания веков, рекуперируя кинетическую в потенциальную и обратно.
Никакие законы физики не запрещают 100% КПД. И тем более не запрещают ассимптотически к нему приближаться.

>да и кинетическая энергия необратимо рассеялась в виде тепла
Нет не рассеялась. Гугли ретардер и не путай его с тормозом на трении. Ретардер это, грубо говоря, когда мы заставляем транспорт за счёт своей инерции взводить пружинку. Вместо трения в колодках с выделением тепла получаем взведённую пружинку, которую мы потом можем использовать для повторного разгона.
Аноним 28/10/19 Пнд 08:07:33 525557222
>>525556
>Это с каких таких хуёв?
С законов физики. Твой поезд едет, энергия переходит в тепло в колесах, энергия уже потеряна, твой поезд тормозит, энергия преобразуется в электрическую, КПД электрогенератора не 100 процентов, он греется, энергия переходит в тепло. И т.д.
Аноним 28/10/19 Пнд 08:11:08 525558223
>>525556
>взводить пружинку.
Совершается работа, тело нагревается и тепловая энергия рассеивается в окружающую атмосфэру.
Аноним 28/10/19 Пнд 10:59:50 525566224
Если солнце полностью накрыть зеркалом, которое разрушится только при температуре 100кк градусов, разрушится ли оно?
Аноним 28/10/19 Пнд 11:59:14 525577225
>>525566
Нагреется и переизлучать обратной стороной начнет. Всего при шести тысячах градусов в нулевой баланс выйдет.
Аноним 28/10/19 Пнд 12:09:21 525580226
Bate2019Metal00[...].mov (19717Кб, 1280x800, 00:00:41)
1280x800
Bate2019Metal1T[...].mov (15936Кб, 1280x800, 00:00:55)
1280x800
Анон, поясни откуда такие различия в температуре газа (во внешних областях в начале оседания кластера) при различном содержании металлов? Чем это обусловлено? В источнике про это толком ничего не написано. Первое видео: 1/100 металличности солнца, второе - солнечное значение. Масса кластера 500 масс солнца, диаметр в начале 2,6 световых года

Аноним 28/10/19 Пнд 12:17:10 525582227
>>525577
Это зеркало, 100% отражение во всём диапазоне, ну понятно из контекста же должно быть, что раз задача гипотетическая, то инженерно-материаловедческими особенностями пренебречь нужно.
Аноним 28/10/19 Пнд 12:33:59 525584228
>>525582
Так оно конвекцией от солнечной атмосферы нагреется, и протуберанцами. Или ты его за орбитой Плутона хочешь поставить? Ну так внешние слои гелиосферы перегреются, атомы наберут скорость убегания и все равно заполнят весь шарик, сожрав планеты по пути. Плазма будет хоть и разряженная, но горячая.
Аноним 28/10/19 Пнд 13:37:26 525589229
>>525512
> Рано или поздно появятся всякие аттосекундные гамма-лазеры
Ну и будешь ты делать аттометровые червоточины, схлопывающиеся в течение аттосекунд, лол.
Аноним 28/10/19 Пнд 13:40:55 525590230
>>525556
> Это с каких таких хуёв? Возьми маятник в вакууме на магнитном подвесе, и будет он у тебя дрочиться до скончания веков, рекуперируя кинетическую в потенциальную и обратно.
Это ты где такой идеальный вакуум возьмешь? Такого не бывает, маятник остановится из-за влияния внешних магнитных полей.
Аноним 28/10/19 Пнд 13:48:10 525592231
Аноним 28/10/19 Пнд 13:53:56 525593232
>>525582
>Это зеркало, 100% отражение во всём диапазоне
Но отражает в электромагнитном спектре, а что делать с потоком ядер водорода и гелия? Они будут соударяться с зеркалом и передавать ему свою энергию -- зеркало начнет нагреваться.
Аноним 28/10/19 Пнд 16:17:02 525614233
>>525543
>Курс доллара, евро, биткоина и йены на декабрь 2020 года, пожалуйста
Ты совсем уебан? Мы 50 лет не можем ничего сильнее, чем полёт на луну сделать, при чем тут доллар?
Аноним 28/10/19 Пнд 16:21:22 525616234
>>525614
>Ты совсем уебан? Мы 50 лет не можем ничего сильнее, чем полёт на луну сделать
У нас уже четыре марсохода по марсу погоняли и прямо сейчас на орбите Марса висят наши спутники и скорость передачи данных Земля-Марс сейчас до 2 Мбит/с.
Такие дела.
Аноним 28/10/19 Пнд 23:31:15 525684235
>>525584
>Так оно конвекцией от солнечной атмосферы нагреется
Ты так, блядь, говоришь, будто оно у тебя прямо с фотосферой лижется. Пока на 0.5 ае оно нагреется, оно сто раз упасть успеет (ибо нестабильно)
Аноним 28/10/19 Пнд 23:42:09 525686236
>>525535
>Звезды немедленно погаснут, потому что термоядерные процессы станут невозможны
Так, падажжи! Схуя ли бы это ТЯ рекции станут невозможны, там сильные и электромагнитные роляют хотя вот в протон-протонном цикле что-то такое должно быть
А светить они в любом случае ещё миллионы лет будут, пока не остынут
Аноним 29/10/19 Втр 01:48:30 525699237
>>525684
Это у тебя сейчас водород и гелий остывают и падают обратно, создавая в этом бурлении фотосферу. Если зеркальным колпаком накрыть, остывать они не будут, и иметь причины падать тоже, так что фотосфера расширится до стенок этой кастрюли.
Аноним 29/10/19 Втр 02:57:24 525703238
>>525542
Квадратный километр паруса из фольги 10 микрон толщиной имеет массу 30 тонн и тягу у Земли 9 ньютонов. Тогда за сутки парус разгонится на v=at=26м/с, и удалится от старта на s=(at^2)/2=1120 км. Все бы ничего, но тяга падает пропорционально квадрату удаления от Солнца, и уже на орбите Марса упадет более, чем вдвое. То есть движение не равноускоренное, а это уже матан с дифурами, хотя и простейший. Но я уже забыл все.
Аноним 29/10/19 Втр 03:22:03 525705239
>>525703
Какие микроны, ты о чем? Паруса уже делают толщиной 20 нанометров.
Аноним 29/10/19 Втр 04:10:56 525710240
>>525705
>Паруса уже делают толщиной 20 нанометров.
Ты уверен, что оно вообще непрозрачно? И какая там прочность, оно эти девять ньютонов выдержит вообще?
Аноним 29/10/19 Втр 04:37:19 525721241
Аноним 29/10/19 Втр 05:45:55 525727242
Аноним 29/10/19 Втр 11:13:53 525752243
А могут ли какие нибудь объекты во вселенной и вправду, хотя бы в теории генерить водород?
Ведь он как-то образовался и без восстановления из более сложных элементов например при электролизе воды в начале существования вселенной, соответственно такой процесс, создания водорода из "ничего" реален.
Аноним 29/10/19 Втр 11:26:19 525757244
Аноним 29/10/19 Втр 12:15:39 525764245
>>525757
Насчет времени начала образования вселенной то понятно, я имел ввиду, есть ли сейчас где-то условия которые позволяют создавать материю из кварков и прочего? То же испарение черных дыр, или допустим бета+ излучение сталкивается с гамма лучами и образуют простейший атом - водород.
Аноним 29/10/19 Втр 12:16:08 525765246
>>525752
Не из ничего, а из эфира. Эфир дохуя подвижен, поэтому в нем легко появляются всякие флуктуации, поэтому мощные потоки эфира, такие как в рукавах галактик и особеноо в центрах галактик, могут стать источником вещественных частиц.
Аноним 29/10/19 Втр 12:40:21 525767247
>>525765
А теплород откуда берётся?
Аноним 29/10/19 Втр 12:45:58 525768248
>>525764
Нет, вряд ли что-то подобное возможно в современной вселенной, кварковой материи нет нигде.
Аноним 29/10/19 Втр 13:04:26 525770249
>>525764
>я имел ввиду, есть ли сейчас где-то условия которые позволяют создавать материю из кварков и прочего?

Можно еще интересней делать, можно создавать целые Вселенные.

Есть гипотеза что можно создавать Вселенные с заданными физическими константами и наша Вселенная одна из бесконечного множества Вселенных с самыми разными константами. И процесс создания не подразумевает что для этого нужно обладать божественной силой. Для этого всего лишь нужна черная дыра. :3
Самым простым способом ее создать будет сталкивать лазерные лучи, так как фотонам наплевать на принцип запрета Паули (фотон является бозоном, а не фермионом), и любое их количество может занимать одно и то же место в пространстве одновременно. Лучше всего использовать гамма-излучение. Если испустить достаточное количество фотонов с таким расчётом, что они одновременно прибудут в точку назначения, в этой точке может сформироваться чёрная дыра.
Это идея получила название Кугельблиц.

А дальше начинается интересное - вблизи сингулярностей черных дыр могут рождаться новые вселенные.

Вселенная уровень 1: миры за пределами нашего космологического горизонта (внеметагалактические объекты).
Вселенная уровень 2: миры с другими физическими константами (например, миры на других бранах в M-теории).
Вселенная уровень 3: миры, возникающие в рамках многомировой интерпретации квантовой механики.
Вселенная уровень 4: конечный ансамбль (включает все вселенные, реализующие те или иные математические структуры).

Гугли "Макс Тегмарк", "Ли Смолин", "Луис Крейн", "Шоун Вестморленд".
Аноним 29/10/19 Втр 13:22:41 525771250
>>525767
Теплород заменен на теплоту. Формулы остались те же и смысл почти тот же.
Аноним 29/10/19 Втр 13:49:21 525775251
>>525770
Квантовый верун, ты?
Аноним 29/10/19 Втр 13:51:26 525776252
>>525770
>Если испустить достаточное количество фотонов с таким расчётом, что они одновременно прибудут в точку назначения, в этой точке может сформироваться чёрная дыра.
И сразу отмазка - "достаточное". Вы недостаточно молились, чтобы пошел дождь. Неси количество, или я тебя зажарю концентрированными в фокусе лупы фотонами, как муравья. Некоторые прилетят одновременно.
Аноним 29/10/19 Втр 13:53:06 525778253
>>525771
Вот этот в теме, и уважвет старших, кросавчег!
Аноним 29/10/19 Втр 14:05:49 525781254
malin.jpg (38Кб, 259x194)
259x194
>>525775
>Квантовый верун, ты?
Аноним 29/10/19 Втр 14:19:09 525782255
>>525776
Радиус Шварцшильда тебе в помощь. Из него можно вывести сколько нужно энергии-массы в некоторой фиксированной точке пространства, чтобы оная энергия-масса сколлапсировала в черную дыру.

Аноним 29/10/19 Втр 14:54:17 525797256
>>525768
За границами вселенной жи есть кварковые флуктуации.
Аноним 29/10/19 Втр 14:59:44 525802257
>>525797
За границами вселенной и цветные лошади есть.
Аноним 29/10/19 Втр 15:36:14 525809258
>>524797 (OP)
Что такое теория струн? Типа мир состоит из таких струн? А почему именно струн, а не из пирожков или балалаек? Это какая-то хуйня типа плоской Земли?
Аноним 29/10/19 Втр 15:44:20 525817259
>>525797
На границе вселенной "хуй" написано, а за ней дрова лежат.
Аноним 29/10/19 Втр 15:46:11 525818260
Аноним 29/10/19 Втр 17:03:58 525833261
>>525781
Я тебя примерно так и представлял. Насмотрятся своей матрицы, а потом начинают верить во всемогущество квантовой физики.
Аноним 29/10/19 Втр 23:17:30 525909262
На Марсе погодка днем то норм всего -15

Если бы еще атмосферу получше

Сейчас я так понимаю там нечем дышать?
Аноним 30/10/19 Срд 01:03:19 525924263
>>525909
>На Марсе погодка днем то норм всего -15

Средняя температура на Марсе −63°С. При наиболее благоприятных условиях летом на дневной половине планеты воздух прогревается до 20°С (а на экваторе — до +27°C) . Но зимней ночью мороз может достигать даже на экваторе от −80°C до −125°С, а на полюсах ночная температура может падать до −143°C.
Аноним 30/10/19 Срд 01:05:17 525926264
>>525924
Ну видишь, то норм, то не норм. По-всякому.
Аноним 30/10/19 Срд 01:06:55 525927265
>>525926
>то норм
Если для тебя −63°С это норм, то ок.
Аноним 30/10/19 Срд 08:55:26 525955266
Аноним 30/10/19 Срд 10:13:01 525960267
Человеку страшно падать в чёрную дыру, потому что это неизведанное?

Но ведь что страшнее, упасть в чд или умереть в аварии. Мне кажется второе гораздо страшнее, а чд чём там, просто сдохнешь и всё, быстро, чётко, без страданий.
Аноним 30/10/19 Срд 10:28:05 525963268
>>525960
>Человеку страшно падать в чёрную дыру
Человек еще ни разу не падал в нее и очень может быть что никогда и не упадет, уд слишком они далеко.
Аноним 30/10/19 Срд 10:42:49 525970269
>>525960
Не страшнее чем ходить по струнам, лазать по бранами, пролетать в червоточинах, перемещаться назад во времени, расширяться вместе с вселенной, наблюдать термодинамическую смерть вселенной, находиться в суперпозиции как кот шредингера или в связанном состоянии как фотоны, тунелироваться сквозь твердую стену с некой верояностью, набирать бесконечную энергию при достижении скорости света. Хуле страшного в маняфантазиях? Это же только фантазии.
Аноним 30/10/19 Срд 10:49:11 525971270
>>525960
>просто сдохнешь и всё, быстро, чётко, без страданий
Ну хуй знает, разрываться от плавно, но неуклонно нарастающих приливных сил должно быть неприятно. Средневековым эквивалентом этого была пытка на дыбе.
Аноним 30/10/19 Срд 11:02:57 525976271
Почему в Луну не врезаются большие астероиды? Сколько нужно ждать чтобы по Луне ударилось что-нибудь создав кратер типа Тихо? какой яркой будет вспышка от тако импакта?
Аноним 30/10/19 Срд 11:17:50 525977272
>>525976
> Почему в Луну не врезаются большие астероиды? Сколько нужно ждать чтобы по Луне ударилось что-нибудь создав кратер типа Тихо? какой яркой будет вспышка от тако импакта?
Вангую, что могло упасть уже упало, а рандомные астероиды крайне редко летают.
Аноним 30/10/19 Срд 12:58:33 525991273
>>525699
>Если зеркальным колпаком накрыть, остывать они не будут
Ты представляешь, каким образом они остывают? Ты вообще понимаешь, что отдельные частицы в глубоком вакууме летают по законам орбитальной механики, а не термодинамики?
Аноним 30/10/19 Срд 13:58:13 526003274
>>525991
>отдельные частицы в глубоком вакууме летают по законам орбитальной механики, а не термодинамики
Как что-то противоречивое. Получила частица пинок больше третьей космической, да и полетела. Температуру сам посчитаешь?
Аноним 30/10/19 Срд 14:15:16 526009275
>>525927
Ну это температура Антарктиды.
Аноним 30/10/19 Срд 14:17:44 526011276
>>524862
Хз. Я вот тоже не понимаю, схуяли там сингулярность должна быть.
Вот взять слияние черных дыр. Это что тогда получается, одна сингулярность падает в другую что ли? Бред какой то.
Аноним 30/10/19 Срд 14:22:24 526014277
>>526011
>Хз. Я вот тоже не понимаю, схуяли там сингулярность должна быть.
Эту ахинею с сингулярностями астрофизикам подкинули математики и астрофизики на нее клюнули. Когда математик не может свои долбанные уравнения решить, то он все валит на сингулярность, дескать тут ничего решить нельзя, возникает ебаная сингулярность.
Экспериментально создать черную невозможно, наблюдений ее тоже нет, так что остается строить гипотезы, некоторые из которых откровенно псевдонаучные, такие как наличие некой "сингулярности" внутри черной дыры.
Аноним 30/10/19 Срд 14:30:03 526020278
>>526014
>математик не может свои долбанные уравнения решить
Мань, это уравнения такие, а не математик такой. А то, что уравнения такие, а не другие, - проблема (или не проблема) физической модели. Если модель соответсвует действительности, значит сингулярность есть. Если не соответствует - наверное нет. Нельзя те же уравнения решить по-другому, можно только заменить уравнения, создав другую модель. Только вот пока ничего столь же точно соответствующего действительности не создали.
Аноним 30/10/19 Срд 14:33:20 526024279
>>526020
Ну вот, пускай и меняют, а то бред же получается.
Аноним 30/10/19 Срд 14:35:38 526026280
>>526020
>Мань, это уравнения такие
Уравнения пишут ученые и если математику простительно что в кго уравнении есть точка, в которой функция стремится к бесконечности, то астрофизику перенимать идею математической сингулярности эти значит выставить себя на посмешище, показать, что ты не знаешь природу черной, но ты уверен, что там сингулярность.
Аноним 30/10/19 Срд 14:36:54 526027281
>>526020
>Если модель соответсвует действительности,
Так в том то и проблема, что она не соответствует экспериментальным наблюдениям. И ты прекрасно знаешь о чем я иначе что ты забыл в спейсаче?
Аноним 30/10/19 Срд 14:41:23 526028282
>>526020
>Если модель соответсвует действительности, значит сингулярность есть. Если не соответствует - наверное нет.
Не так. Вообще говоря твой оппонент пытался выразить мысль, что та область пространства где сингулярность(и вокруг ее) выходит из области определения физмодели / соответствия действительности.

>Нельзя те же уравнения решить по-другому, можно только заменить уравнения, создав другую модель.
Это да.
Аноним 30/10/19 Срд 14:46:17 526033283
>>526028
>выходит из области определения физмодели
Да собственно вся Вселенная, а не только лишь черные, вопит нам, что наша модель ошибочна.
Аноним 30/10/19 Срд 14:57:29 526039284
>>526028
Я доебался именно к термину "математики". Математики не придумывают физические модели, математика только разрабатывает аппарат для них. С точки зрения математики, все модели одинаково хороши, даже какой-нибудь плоской Земли, лишь бы они были внутренне непротиворечивы. Насколько уж они соответствуют реальности - это не их забота.
Аноним 30/10/19 Срд 14:57:32 526040285
Поясните за гиродин, это везде в космосе применяется? Оно много энергии жрёт?
Аноним 30/10/19 Срд 15:04:17 526044286
>>526027
Неси эксперименты, опровергающие ОТО.
30/10/19 Срд 15:10:30 526047287
>>526024
>- Алло, это астрофизика? Поменяйте уравнения, а то бред же получается.
>- Сообщите нам, пожалуйста, уравнения без бреда.
>- Кто? Я?
Аноним 30/10/19 Срд 15:12:52 526049288
>>526044
Гугли Dark Energy, Dark Matter.
Аноним 30/10/19 Срд 15:18:43 526051289
>>526049
Я думал, ты что-то интересное принесешь, а ты в штаны навалил зачем-то. Что темная материя, существование которой практически 100% подтверждено, что темная энергия, где пруфы еще не железобетонные, с ОТО полностью совместимы и наблюдениям соответствуют.

В следующем посте жду конкретное разъяснение, как именно ОТО «не соответствует экспериментальным наблюдениям», или с тобой и разговаривать нет смысла.
Аноним 30/10/19 Срд 15:29:42 526057290
>>526047
>Сообщите нам, пожалуйста, уравнения без бреда.

Легко. Внутри дыры метрика де Ситтера-Шварцшильда и вуаля, никакой ебаной сингулярности!
Аноним 30/10/19 Срд 15:31:18 526059291
>>526051
>ты что-то интересное принесешь
А тебе не интересно, что Вселенная состоит из чего-то что мы не знаем и это самое никак не учтено в нашей модели? По-моему это чертовски интересно всем, кто интересуется космологией.
Аноним 30/10/19 Срд 15:58:21 526066292
>>526059
>никак не учтено в нашей модели
Ты даун или прикидываешься? Ты в курсе вообще, как расшифровывается название основной текущая модель вселенной, ΛCDM?

Лямбда — космологическая постоянная (темная энергия).
CDM — cold dark matter (темная материя).

ΛCDM полностью совместима с ОТО, более того, сама лямбда и появилась изначально в уравнениях ОТО как энергия вакуума, а экспериментально она обнаружена была только через 50+ лет. Никак не учтено в нашей модели, охуеть.
Аноним 30/10/19 Срд 15:59:08 526067293
>>526051
>с ОТО
В решениях уравнений Эйнштейна есть ебаные сингулярности, что автоматически означает, что это херня на постном масле, а не теория. Одним из главных критериев объяснительной силы теории является предсказательная сила, а когда в теории вводится ебаная сингулярность, то ни о какой предсказательной силе речи идти не может.
Аноним 30/10/19 Срд 16:05:38 526071294
>>526067
Ты же просто нейросеть, которая случайный текст высирает по ключевым словам, ты даже не пытаешься в диалоге участвовать.

Разговор начался с твоего заявления, что ОТО «не соответствует экспериментальным наблюдениям», но ничего подобного в треде до сих пор не появилось. Пока не принесешь ссылки на опровергающие ОТО эксперименты, причем проведенные не только внутри твоей головы, говорить больше не о чем.
Аноним 30/10/19 Срд 16:07:13 526074295
>>526066
>текущая модель вселенной, ΛCDM

Лол, отличная модель, которая не обладают никакой объяснительной силой. Какова природа темной материи? Эта модель ничего об это не знает. Какова природа темной энергии? Эта модель ничего об это не знает. Что находится в черной дыре? Эта модель ничего об это не знает, поэтому утверждает, что там некая сингулярность. Отличная модель, да.
Аноним 30/10/19 Срд 16:23:25 526077296
>>526067
Дебил обоссанный, нихуя не понимаешь в том, о чём пытаешься пиздеть. Я вот тоже не верю в сингулярности (это именно вопрос веры), но ОТО напредсказывала столько, что в пизде твоей мамаши едва ли уместится.
Аноним 30/10/19 Срд 16:28:44 526081297
>>526074
>Какова природа темной энергии? Эта модель ничего об это не знает
Теория эволюции тоже ничего об этом не знает, следовательно она неверна.
Аноним 30/10/19 Срд 19:27:14 526163298
>>525927

Почти как в Якутске зимой хуле

Модно жить
Аноним 30/10/19 Срд 19:52:06 526167299
>>526077
Дублирую этого квантопоклонника.
Аноним 30/10/19 Срд 21:26:30 526188300
>>526167
У вас церковь квантовопоклонников, да?
Аноним 30/10/19 Срд 23:45:45 526224301
>>526077
ОТО обоссывается на малых размерах. А сингулярность как раз из такого.
Аноним 31/10/19 Чтв 01:42:39 526257302
>>526224
>сингулярность
Бесконечно маленькая точка в которой значение функции стремится к бесконечности т.е. точка в которой предсказания модели становятся бессмысленными. Идея сингулярности - это самый большой фейл физиков, это признание их неспособности построить эффективную модель.
Аноним 31/10/19 Чтв 01:46:04 526260303
///
Аноним 31/10/19 Чтв 01:53:09 526267304
>>526257
Какова плотность элементарной частицы, у которых, по современным представлениям, нулевой размер? Разве плотность массивной точки не бесконечна?
Аноним 31/10/19 Чтв 02:06:23 526277305
>>526267
>Какова плотность элементарной частицы
А нет никаких "элементарных частиц". Это все возбуждения различных полей, а вместо волновой функции есть полевой оператор. :3

Аноним 31/10/19 Чтв 09:18:33 526349306
1431762original.png (785Кб, 1000x620)
1000x620
Почему аппараты для лунной миссии выводились на орбиту цельными?
Неужели никак не получится вывести: лунный модуль, возвращаемый модуль и разгонный блок до луны, но по отдельности на трех ракетах?
Ведь эти модули стыкуются и расстыковываются, а стыковка вроде не такое невероятное мероприятие на орбите?
Аноним 31/10/19 Чтв 10:23:17 526360307
растягивание.png (12Кб, 569x654)
569x654
сжатие.png (12Кб, 664x400)
664x400
сжатия нет.png (16Кб, 1049x615)
1049x615
rings.jpg (36Кб, 660x471)
660x471
>>524797 (OP)
Приливных сил вопрос: как действуют приливные силы на тело,

1. находящееся на круговой орбите
2. двигающееся с гиперболической скоростью в центральном поле тяготения

Мне раньше казалось, что приливные силы всегда сдавливают тело с боков и растягивают вдоль направления действия силы, безотносительно движения тела. Но вот если тело находится на круговой орбите, то в плоскости орбиты приливные силы такое ощущение что сдавливать не будут, только перпендикулярно её плоскости. Так или это, или всё же нет?

И не этим ли задана форма планетных колец?
Аноним 31/10/19 Чтв 10:37:06 526364308
>>526257
Дебил клинический.
Аноним 31/10/19 Чтв 10:54:12 526365309
>>526349
Потому что решили, что так будет лучше и экономичнее. Рассматривали и схему с несколькими стартами, и сейчас рассматривают. Но вообще, она выгоднее только если разработка более тяжёлой ракеты слишком сложна или дорога.
Аноним 31/10/19 Чтв 11:10:51 526366310
>>526267
>у которых, по современным представлениям, нулевой размер
У них неизвестно какой размер, может быть планковский, или размер скрученной струны. Очевидно, что ОТО на таком масштабе неприменима, иначе бы все элементарные частицы были чёрными дырами, вот и получается, что сингулярности скорее всего не существуют. Это не исключает существования чёрных дыр, а также не уменьшает ценности ОТО в своих границах применимости.
Аноним 31/10/19 Чтв 14:07:36 526407311
>>526365
>если разработка более тяжёлой ракеты слишком сложна
Но ведь N1 не довели до ума, могли бы и раскошелится на два лишних Союза, раз сверхтяжелые не получаются.
Олсо, это ещё спорный вопрос что дешевле запустить, одну N1 с дохуилярдом движков-прототипов, почти ручной работы или три Союза, двигатели которых штампуются в путь.
Аноним 31/10/19 Чтв 14:15:37 526408312
>>526360
>приливные силы всегда сдавливают тело с боков и растягивают вдоль направления действия силы
Они никого не сдавливают. Сдавливает самогравитация тела. На центр масс тела действует сила Fc, на точку, ближайшую к большому телу - Fa, на самую удаленную - Fb. За счет возрастающего расстояния от большого тела, Fa > Fc > Fb. То есть, если смотреть от центра масс, в точках a и b действуют силы (Fa - Fc) > 0 и -(Fb - Fc) > 0, направленные против самогравитации.

То есть, в норме малое тело всюду равномерно "обжимается" самогравитацией. А в присутствии приливных сил возникают силы, действующие против самогравитации, причем они максимальны в точках максимально приближенных и максимально удаленных от большого тела. Поэтому, тело начинает пучить в этих направлениях, потому что с других сторон самогравитация давит как и раньше.

Для грубой аналогии, возьми мягкий мячик и сдави его "по экватору". На полюсах его ничего не тянет, но выпячиваться там он все равно начнет.
Аноним 31/10/19 Чтв 14:17:47 526409313
>>526408
>Они никого не сдавливают. Сдавливает самогравитация тела.
Нет, тут ты не прав. В самом грубом случае - космонавта, падающего в черную дыру, спагеттизирует вовсе не его самогравитация.
Аноним 31/10/19 Чтв 14:19:17 526411314
>>526409
При чем тут спагеттизация к сдавливанию вообще?
Аноним 31/10/19 Чтв 14:29:12 526412315
image.png (3Кб, 160x312)
160x312
>>526411
Потому что спагеттизация осуществляется приливными силами. Независимо от массы разрываемого объекта.

С боков сдавливание идет, т.к. направления силы тяжести не параллельны друг другу, а сходятся к общей точке. Вдоль разрывание идет, т.к. снизу тянет сильнее, чем сверху.
Аноним 31/10/19 Чтв 14:34:38 526413316
>>526412
Ладно, сдавливание есть, но оно на порядки слабее обычного проявления приливных сил. То есть, |Fсдавливания| << |Fa - Fc|.

И на спагеттификацию оно не влияет никак. Если предположить, что направления строго параллельны, спагеттификация все равно произойдет ровно с тем же успехом. Она зависит только от |Fa - Fc|.
Аноним 31/10/19 Чтв 14:38:08 526415317
>>526413
> |Fсдавливания| << |Fa - Fc|
Такое надо обосновывать расчётами.
Аноним 31/10/19 Чтв 14:40:07 526416318
image.png (49Кб, 659x344)
659x344
>>526413
>Ладно, сдавливание есть, но оно на порядки слабее обычного проявления приливных сил. То есть, |Fсдавливания| << |Fa - Fc|.
Блин зачем ты так безапелляционно утверждаешь ерунду? И вообще не об этом речь.
Аноним 31/10/19 Чтв 14:44:27 526417319
>>526415
Ща прикинул, порядок один, да (получилось и то, и то ~ r / R^3, где r - радиус малого тела, R - расстояние до большого). Так что я был не прав. Правда, на спагеттификацию все равно в первую очередь растягивание влияет, а не сдавливание.
Аноним 31/10/19 Чтв 15:00:03 526420320
>>526415
>>526417
Посчитал для Земли, получилось растягивание примерно вдвое сильнее сдавливания. Т.е. порядок один.

>Правда, на спагеттификацию все равно в первую очередь растягивание влияет, а не сдавливание.
Интересный вывод, лол. Ладно, вопрос не об этом.

Вопрос - выключается ли это сдавливание в плоскости орбиты? Мне кажется, что да.
Аноним 31/10/19 Чтв 15:03:20 526421321
>>526420
>>Правда, на спагеттификацию все равно в первую очередь растягивание влияет, а не сдавливание.
>Интересный вывод
Если бы растягивания не было, то тело бы не спагеттифицировалось. Человек, например, просто сжался бы в шарообразную фрикадельку повышенной плотности, если бы его ничто не тянуло в стороны.
Аноним 31/10/19 Чтв 15:07:07 526424322
>>526420
>выключается ли это сдавливание в плоскости орбиты?
Ну, я уже один раз обосрался, но не вижу с чего бы. Точно также притяжение крупного тела идет под небольшим углом, и при вычитании векторов получится результат, идущий к центру.
Аноним 31/10/19 Чтв 15:08:10 526426323
stan.jpg (284Кб, 1200x780)
1200x780
>>526421
>шарообразную фрикадельку
Это как? ))) ИМХО обе силы работают гармонично, обжимая с боков и растягивая вдоль. В случае градиента обычной черной дыры и той, и той силы по одиночке хватило бы с лихвой. При чисто обжатии его бы так же спрессовало в нитку, по сути прокатный стан так и работает.
Аноним 31/10/19 Чтв 15:09:24 526428324
>>526426
Бля, сжатие ж не равномерно со всех сторон. Ладно, сегодня больше не буду постить.
Аноним 31/10/19 Чтв 15:12:22 526432325
>>526417
Прикинул на пальцах на пхытоне.

Для сферического космонавта диаметром 1м и массой 100кг на расстоянии 1000км от ЧД массой в 10 солнечных масс растягивающая сила будет 267кН, а сдавливающая — 133кН.

Вполне можно выбирать, от чего быстрее хочется умереть, разворачивая нужным способом свою тушку в пространстве.
Аноним 31/10/19 Чтв 15:13:12 526433326
>>524797 (OP)
Как понять парадокс, что для наблюдателя тело падающее на черную дыру будет падать бесконечно долго? Тогда выходит, что если взглянуть на черную дыру, то вокруг нее должен быть сплошной слой вещества которое на нее когда-либо падало?
Аноним 31/10/19 Чтв 15:14:55 526435327
>>526424
Я тоже всегда так думал, но вот похоже что нет. См. мэдскиллз рисунки в моем первом посте. Для неподвижной палки оба направления изотропны, но вот для летящий по орбите - получается что нет. В плоскости орбиты я не вижу приливных сил, порисуй сам.

Представь себе две не связанные точки на одной орбите, летящие друг за другом. Они будут вечно неподвижны и будут занимать одну орбиту. Нет силы, их сталкивающей. Наоборот, если ты попытаешься поставить их рядом, они будут колебаться навстречу друг другу, а если друг под другом - то будут прыгать вверх-вниз.
Аноним 31/10/19 Чтв 15:17:29 526436328
>>526433
>Тогда выходит, что если взглянуть на черную дыру, то вокруг нее должен быть сплошной слой вещества которое на нее когда-либо падало?
Да. Слой, практически бесконечно тонкий и излучающий на практически бесконечной длине волны. А ещё совокупная масса системы ЧД + тело имеет горизонт событий чуть побольше радиусом, и толшина этого размазанного слоя очень быстро становится меньше этой дельты. Т.е. объект за конечное (и небольшое, но лень считать) время все же скрывается под горизонтом событий.
Аноним 31/10/19 Чтв 15:20:31 526438329
> Почему
По той же причине почему на Марс не полетели.
Пропаганда рисует картину космонавтики слабо связанную с реальностью. В тч простоту схемы со стыковкой (особенно в 1960-х) сильно преувеличивают.

>>526349
>Неужели никак не получится вывести: лунный модуль, возвращаемый модуль и разгонный блок до луны, но по отдельности на трех ракетах?
Там нужно больше трех ракет если говорить о протонах тех-же.
Нужно что бы все эти аппараты летали по орбите без критических сбоев...и им хватило срока жизни. То есть та же электроэнергия на апполонах добывалась в топливных элементах (а результирующая вода выпивалась космонавтами) и срок жизни корабля в космосе был очень небольшой. При этом срок подготовки ракеты к старту весьма и весьма долгий.... значит нужно несколько стартовых площадок и так далее.

>Почему аппараты для лунной миссии выводились на орбиту цельными?
>Ведь эти модули стыкуются и расстыковываются, а стыковка вроде не такое невероятное мероприятие на орбите?
В 1969 году это было именно что невероятное мероприятие на орбите. В первую попытку пилотируемой стыковки ссср вообще не смог, в отличие от сша. После чего переделывал союз.
Аноним 31/10/19 Чтв 15:26:03 526444330
>>526436
>но лень считать)
А какие формулы для этого нужны?
Аноним 31/10/19 Чтв 15:33:18 526449331
>>526438
>После чего переделывал союз.
Хотя не, прогнал. Вроде не переделывал.
Аноним 31/10/19 Чтв 15:48:10 526455332
>>526444
Надо курить ОТО и прочий матан, я где-то находил готовое решение и удовлетворился им. Кстати реальные черные дыры (не гипотетические первичные) по сути такие же остановившиеся во времени гравитационные коллапсы.
Аноним 31/10/19 Чтв 15:53:21 526460333
>>524797 (OP)
У фотонов в вакууме скорость 300 000 км/с. А какая скорость у фотонов в стали?
Аноним 31/10/19 Чтв 15:54:11 526462334
>>526455
>остановившиеся во времени
В каком смысле?
Аноним 31/10/19 Чтв 15:56:14 526463335
>>526462
В том смысле что твои рассуждения о том, что в черную дыру нельзя упасть с т.з. внешнего наблюдателя, полностью верны и для самого образования ЧД в результате коллапса звезды. Падающее вещество звезды замирает за миг до. Навсегда.
Аноним 31/10/19 Чтв 16:06:10 526465336
>>526463
>Падающее вещество звезды замирает за миг до
А ты уверен, что правильно онимаешь ОТО?
Аноним 31/10/19 Чтв 16:14:23 526467337
>>526465
Мне кажется да. Что не так?
Аноним 31/10/19 Чтв 16:20:29 526470338
>>526465
"Кроме того, чёрными дырами часто называют объекты, не строго соответствующие данному выше определению, а лишь приближающиеся по своим свойствам к такой чёрной дыре — например, это могут быть коллапсирующие звёзды на поздних стадиях коллапса. В современной астрофизике этому различию не придаётся большого значения[7], так как наблюдаемые проявления «почти сколлапсировавшей» («замороженной») звезды и «настоящей» («извечной») чёрной дыры практически одинаковы. Это происходит потому, что отличия физических полей вокруг коллапсара от таковых для «извечной» чёрной дыры уменьшаются по степенным законам с характерным временем порядка гравитационного радиуса, делённого на скорость света — то есть за доли секунды для чёрных дыр звёздных масс и часы для сверхмассивных чёрных дыр[8]."

Википедия
Аноним 31/10/19 Чтв 16:26:03 526472339
Аноним 31/10/19 Чтв 16:28:49 526475340
Анон, поясни за детонационник. В чем профиты, в чем отличия от обычного жрд?
Аноним 31/10/19 Чтв 16:47:44 526480341
>>526040
Применяется практически везде, даже на кубсатах, связано с прогрессом в области электродвигателей с редкоземельными магнитами. Диды использовали вращение всего корпуса аппарата, аэродинамическую (солнечными панелями) и гравитационную (длинная станция перпендикулярно орбите одним концом на Землю) стабилизации. Или мирились с постоянным вращением, подготовка космонавтов включает это в себя. Стыковка без гиродинов пока невозможна, рулевые двигатели имеют квантованный выхлоп.
Аноним 31/10/19 Чтв 16:55:07 526482342
>>526475
Профитов много. Чтобы обычный ЖРД был годным, надо сжать газообразные компоненты до нескольких сотен атмосфер турбиной с насосом (самый сложный и проблемный агрегат), в детонационнике сжатие делается детонационной волной. Детонация вдобавок позволяет отказаться от критического сечения сопла, это еще одно узкое место ЖРД.
Аноним 31/10/19 Чтв 16:59:37 526483343
>>526433
Да, но при падении тела для внешнего наблюдателя тускнеют и их становится не видно.
Аноним 31/10/19 Чтв 16:59:59 526484344
Аноним 31/10/19 Чтв 17:00:17 526485345
>>526460
Смотря какая длина волны.
Аноним 31/10/19 Чтв 17:03:10 526486346
>>526475
Ключевая идея — вместо поднятия давления топлива турбонасосами пускать по нему детонационную волну. Это позволяет достичь большего давления в камере сгорания и, как следствие, большего удельного импульса, потому что удельный импульс зависит от разницы давлений между камерой и окружающей средой. Бывают импульсные и кольцевые. Кольцевые — это прям ёба из ёб, детонационная волна десятки тысяч раз в секунду обегает кольцевую камеру сгорания, в которую постоянно подаётся свежее топливо. На этом видосе снято в слоумо на лабораторном прототипе, как это происходит:

0:55
https://www.youtube.com/watch?v=pHcxI-8GtZg

Аноним 31/10/19 Чтв 17:21:43 526492347
>>526435
>Представь себе две не связанные точки на одной орбите, летящие друг за другом. Они будут вечно неподвижны и будут занимать одну орбиту. Нет силы, их сталкивающей.
Насколько я понимаю, сила толкающая их друг к другу таки есть. Если рассмотреть эти две точки без центрального тела, можно увидеть, что одна вращается вокруг другой (ну или вторая вокруг первой, раз уж они одинаковы) с тем же периодом, что и вокруг центрального тела. И, как я это понимаю, именно аналог приливной силы толкает одну точку к другой и работает как центростремительная сила, создающая круговую орбиту.
Аноним 31/10/19 Чтв 18:59:22 526503348
>>526492
Чет по-моему ты просто описал обычную силу притяжения, без центрального тела приливных сил не будет по определению.
Аноним 31/10/19 Чтв 19:10:00 526507349
>>526503
"Без центрального тела" я имею в виду если просто на него "не смотреть", а не так чтоб его убрать. Короче, как объясняют, что Луна таки вращается вокруг собственной оси: нужно посмотреть на Луну, не обращая внимание на Землю. Так же и тут, если посмотреть на точки, не обращая внимание на центральное тело, станет видно, что они вращаются друг вокруг друга.
Аноним 31/10/19 Чтв 19:20:35 526513350
>>526507
А, ты про это. Нет, это тоже мимо. Если была бы сила, они бы сближались. Как это происходит в других плоскостях.
Аноним 31/10/19 Чтв 19:23:36 526514351
>>526513
На МКС действует сила, строго направленная на Землю, но она с Землей не сближается.
Аноним 31/10/19 Чтв 19:37:38 526519352
>>526514
Хм, надо подумать. Для колебаний по двум другим осям период колебаний тоже будет точно равен одному обороту по орбите. Но в них меняется расстояние между точками.
Аноним 31/10/19 Чтв 19:46:58 526524353
>>526514
Вот какая-то научная работа на похожую тему:
http://www.jetp.ac.ru/cgi-bin/dn/r_153_0232.pdf

Чтобы вникать на таком уровне в матан я все забыл. Однако там есть утверждение:

>3. ПРИЛИВНЫЕ СИЛЫ
>Существование приливных сил связано с концепцией расхождения геодезических, вдоль которых движутся частицы.

Точки на одной орбите имеют одну геодезическую линию, собственно орбита и является геодезической линией.
Аноним 01/11/19 Птн 00:52:46 526655354
>>526484
Как так? Фотоны не могут пролетать сквозь сталь?
Аноним 01/11/19 Птн 05:24:34 526708355
1) Почему если температура - это скорость с которой движутся молекулы, предметы с кристаллической решеткой не кажутся супер холодными?
2) Почему предметы с кристаллической решеткой не кажутся чуть холоднее чем жидкости и газы, в которых скорость молекул всяко выше?
Аноним 01/11/19 Птн 05:41:50 526709356
>>526708
>Почему если температура - это скорость с которой движутся молекулы

Температура - это энергия теплового движения частиц вещества, приходящегося на одну степень свободы.

>предметы с кристаллической решеткой не кажутся супер холодными?

Потому что даже в кристаллической решетке у частиц есть некоторые степени свободы. Следовательно кристалл может нагреться до температуры окружающей среды.

Аноним 01/11/19 Птн 09:19:14 526738357
Надеюсь, я в правильном треде.

>Прошлый тред провалился в лавовую трубку и потерял связь по этим координатам
>Предыдущий тред опозорился и прогорел вон там
>Первый тред был сведен с орбиты и сгорел в атмосфере Юпитера
>Предыдущий тред попытался перемешать кислород в баке
>Прошлый тред закрыли из-за перерасхода средств вон там
Это один человек придумывает или разные?
Аноним 01/11/19 Птн 10:08:04 526750358
>>526738
Разные, одно из этих моё например.
Аноним 01/11/19 Птн 16:59:08 526837359
Прочитал на Википедии о том, что в созвездии Большой Медведицы есть галактики. Как это понять?
СБМ находится вне Млечного Пути или как?
Аноним 01/11/19 Птн 17:01:31 526839360
>>526837
Это указания зрительного местоположения.
Как сектора на небе.
В Волосах вероники галактик вообще жопой жуй.
Аноним 01/11/19 Птн 17:09:28 526842361
>>526839
Она причесаться не пробовала?
Аноним 01/11/19 Птн 17:19:03 526849362
>>526839
Я просто конченный даун, вообще нихуя не понимаю.
Так, созвездие - это просто группа звёзд, которая при визуальном наблюдении с Земли похожа на некий объект, верно?
Звёзды в созвездии находятся на огромном расстоянии друг от друга и лежат в разных... плоскостях, что ли?
А теперь вопрос: где там галактики?
Аноним 01/11/19 Птн 17:21:18 526850363
>>526849
Просто в направлении этих звёзд.
Аноним 01/11/19 Птн 17:32:30 526852364
>>526849
Созвездие - это группа звезд и область на небе, включающая эти звезды. "Находится в созвездии" подразумевает именно область, или, по-другому, направление на объект от наблюдателя (Земли).
Аноним 01/11/19 Птн 18:01:59 526854365
>>526849
>Так, созвездие - это просто группа звёзд, которая при визуальном наблюдении с Земли похожа на некий объект, верно?
Верно.
>Звёзды в созвездии находятся на огромном расстоянии друг от друга и лежат в разных... плоскостях, что ли?
Да. Звёздам вообще до пизды наши представления о созвездиях, они в объёме хуй знает как разбросаны, это наше воображение сводит их в рисунок на плоскости. Если съебать на Сириус, оттуда все созвездия вообще по-другому будут выглядеть.
>А теперь вопрос: где там галактики?
Галактики настолько охуенно далеко, что их глазом практически не видно. Возьми самую тусклую звёздочку на грани видимости, и примерно так будет выглядеть ближайшая к нам галактика Андромеды - сверхгигантская такая хуйня больше всего Млечного пути. И это ближайшая. Другие галактики в сотни и тысячи раз дальше, без неебического телескопа их вообще не увидеть.

Порядки расстояний растут очень быстро. Если марс лежит у тебя на столе, то ближайшая звезда в другом микрорайоне, а ближайшая галактика в другом городе.
Аноним 01/11/19 Птн 18:46:33 526858366
>>526854
Мы никогда не долетим до других галактик.
Аноним 01/11/19 Птн 19:04:45 526861367
>>526858
Вы и до других звёзд не долетите.
Аноним 01/11/19 Птн 19:06:24 526862368
Аноним 01/11/19 Птн 19:21:20 526863369
Аноним 01/11/19 Птн 20:41:42 526868370
Какова нынешняя скорость расширения пространства? Она измеряется в герцах?
Аноним 01/11/19 Птн 21:11:13 526873371
>>526868
67 км/с на мегапарсек или ~2,2×10−18 с-1. Хотя размерность одна, никаких герц там нет, т.к. расширение пространства это непериодический процесс.
Аноним 01/11/19 Птн 21:13:25 526874372
>>526854
Уточню, что некоторые галактики по угловым размерам в разы больше Луны, а плохо видно их лишь потому, что они очень тусклые.
Аноним 01/11/19 Птн 21:13:34 526875373
>>526873
Спасибо. Про герцы я понимаю, просто так прикольнее.
Аноним 01/11/19 Птн 21:55:13 526894374
>>526873
А если бы скорость была порядка 10^-7 с, чтоб за несколько месяцев любое расстояние увеличивалось вдвое, СС бы распидорасило? Или гравитация все еще смогла бы поддерживать какие-то устойчивые орбиты?
Аноним 01/11/19 Птн 21:55:30 526895375
Аноним 02/11/19 Суб 04:17:12 526980376
mem.jpg (51Кб, 720x534)
720x534
>Тред тупых вопросов
Отлично. Итак:
1) Почему считается, чтобы быстрее скорости света нет ничего?
2) Как может быть, что чёрных дыр не существует? Читал теорию, что их на самом деле нет
2.а) Что за странности в вращающимимися чёрными дыра, которые не точкой, а кольцом?
2.б) Что за заряженные чёрные дыры и откуда у них заряд?
3) Какой минимальный размер должен быть у астероида, чтобы, имя очень большую плотность, а оттого и массу, а значит и тяготение, при малых объёмах тела очень большое, чтобы человеку было бы на нём комфортно жить? Например, завтра находя какой-нибудь планетоид на 220км диаметра но, с гравитацие близкой если не к земной, то к марсовой, будет ли комфортно жить человеку на такой планете? Воздух не улетит? Вода не выпарится из атмосферы? Атмосефра вообще нормальная будет? Предвкушая вопрос о плотности, не знаю, что сказать, предпложим, эта планета состоит из золотоа, свинца и ртути в ядре своём, поэтому большая масса при столь малых объёмах. Естестенно, верхние километров 10 - обычный земной-марсиаский грунт. А хватит ли этого, для водо и воздухо циркуляции? Будет ли у такой планеты вращение и магнитное поле? Оно будет сильнее, интенсивнее, обширнее или что?
4) Поиграв в StarBound, наткнулся там на вечный кислородный баллон, позволяющий дышать и под водой, и на токсичной планете, и в космосе, потмоу что в баллоне растения. И вот я задумался: а сколько должно быть зелени (или, наверное, надо уточнить, каова должная быть общая площадь поверхность листвы и травы как активных дышаших участков растений), чтобы человек мог дышать вечно, выдыхая ровно столько, сколько растения едяь и вдыхая столкьо, сколько нужно человеку, и это именно то количество, которое дают растения?
Аноним 02/11/19 Суб 04:48:49 526984377
>>526980
>Почему считается, чтобы быстрее скорости света нет ничего?
Потому что если ты конвертируешь энергию всей вселенной в разгон, то ты все равно будешь медленнее скорости света. Это эдакий фундаментальный лимит, к которому можно бесконечно приближаться, но невозможно достичь.

Другое дело, что ты можешь гнуть, искривлять и протыкать пространство (см. пузырь Алькубьерре и червоточины). Таким образом ты не превысишь скорость света, ты просто срежешь путь.

Ну а на остальное пусть другие аноны ответят.
Аноним 02/11/19 Суб 07:32:32 526990378
>>526980
>чёрных дыр не существует
Есть разные теории. Чёрные дыры самая запруфанная, консервативная, не требующая изобретения новых сущностей теория.
Но это не мешает теоретикам придумывать новые.
Вот этой особенности науки не понимают зогачеры и кефирщики.
Аноним 02/11/19 Суб 08:55:21 526997379
>>526984
>Потому что если ты конвертируешь энергию всей вселенной в разгон, то ты все равно будешь медленнее скорости света.
Угадай, сколько понадобится энергии, чтобы разогнать фотон до скорости света.
Аноним 02/11/19 Суб 08:57:07 526999380
>>526984
Манипуляции с пространством так то тоже невозможны.
Аноним 02/11/19 Суб 08:58:33 527000381
>>526997
Фотон не нужно разгонять, он всегда движется со скоростью света. Но массивные тела до скорости света разогнать не выйдет.
Аноним 02/11/19 Суб 09:08:55 527002382
>>526980
>1) Почему считается, чтобы быстрее скорости света нет ничего?
Экспериментально установлено, что скорость света не зависит от выбора системы отсчёта, то есть один и тот же фотон летит со скоростью с относительно земли, относительно самолёта, летящего в направлении его движения, и относительно самолёта, летящего против его движения. Чтобы объяснить этот феномен, была предложена Специальная Теория Относительности, согласно которой течение времени зависит от системы отсчёта и одновременность событий зависит от системы отсчёта. С тех пор СТО была многократно экспериментально проверена. Из неё и её математики (достаточно простой, вы пройдёте её в 11 классе), между прочим, следуют некоторые интересные выводы.
1) Чтобы разгонять тело, имеющее массу, нужно тратить всё больше и больше энергии. Чтобы оно достигло скорости света, нужно затратить бесконечное количество энергии.
2) Любые безмассовые частицы в вакууме движутся со скоростью света.
3) Вообще невозможно передать хоть что-то, какую-либо информацию, быстрее скорости света, иначе ты обязательно сможешь передавать и информацию в прошлое, а это нарушает принцип причинности. Неизвестно, как будет дело со всякими червоточинами и пузырями Альбукерке, если они возможны, может быть, перемещённый таким образом звездолёт будет навсегда отрезан от той части вселенной, из которой вылетел.
Аноним 02/11/19 Суб 09:36:10 527005383
>>527000
>Фотон не нужно разгонять, он всегда движется со скоростью света.
А какова скорость света в стали?
Аноним 02/11/19 Суб 12:06:53 527032384
>>526997
Фотон безмассовый, доброе утро. Разгони мне протон до скорости света, пожалуйста. Можешь ещё парочку вселенных заюзать, разрешаю.
Аноним 02/11/19 Суб 12:13:33 527033385
>>527005
Всё ещё C. Или ты спрашиваешь про то, какое расстояние по прямой преодолеет? Так фотоны взаимодействуют путём квантовой электродинамики с другими частицами, от того кажется, что что-то замедляется. Но фундаментально сами фотоны всё ещё движутся со скоростью C.
Аноним 02/11/19 Суб 12:14:36 527034386
Аноним 02/11/19 Суб 12:32:03 527036387
Аноним 02/11/19 Суб 12:36:22 527037388
>>527002
>Экспериментально установлено, что скорость света не зависит от выбора системы отсчёта
Да ти що. Ниче, что как только ты упоминаешь экспериментальное подтверждение, сразу залезаешь обеими ногами в болото статистики и погрешностей?
Аноним 02/11/19 Суб 12:39:06 527039389
advanced-test-r[...].jpg (248Кб, 1948x2560)
1948x2560
>>527005
Излучение Вавилова-Черенкова, здравствуй!
Аноним 02/11/19 Суб 12:45:59 527040390
>>527005
Коэффициент преломления стали 2.5, следовательно ниже в два с половиной раза, чем в вакууме. относится только к телам, размеры которых много больше длины волны
Аноним 02/11/19 Суб 12:52:18 527041391
Аноним 02/11/19 Суб 12:52:56 527042392
>>527032
Если чтото не имеет массы, но переносит энергию, то это волна.
Аноним 02/11/19 Суб 12:55:39 527043393
>>527033
>Всё ещё C.
А сколько в метрах в секунду скорость фотона в стали?
Аноним 02/11/19 Суб 12:56:51 527044394
>>527041
Бля, щас погуглил, Ацюковский 1930 года рождения, жив, и лекции читает. ААААААА!
Аноним 02/11/19 Суб 12:58:48 527045395
>>527039
>Излучение Вавилова-Черенкова,
А при чем тут фотоны? Это результат когда ЗАРЯЖЕННЫЕ РЕЛЯТИВИСТСКИЕ частицы ебашат в воде. Фотон так не умеет.
Аноним 02/11/19 Суб 13:03:19 527046396
>>527040
>Коэффициент преломления стали 2.5, следовательно ниже в два с половиной раз
Хм, почему же мы тогда не видим сквозь сталь?
Аноним 02/11/19 Суб 13:14:25 527048397
>>527037
Дорогой кефирщик, я понимаю, что у вас всё плохо с принятием результатов экспериментов, но чтобы вот так вот отрицать экспериментальную физику вообще? Зачем же тогда кефир? Сразу бога постулируйте.
Аноним 02/11/19 Суб 14:11:30 527061398
>>527043
Сталь поглощает фотоны или отражает их, в зависимости от длины волны. Вопрос некорректен.
Аноним 02/11/19 Суб 14:14:24 527062399
>>527037
>болото статистики
О-ля-ля! Кому это тут бомбит от статфизики?
Аноним 02/11/19 Суб 14:18:10 527064400
>>527061
>Сталь поглощает фотоны
А при "поглощении" фотоны имеют скорость в стали? Во время их великого пробега в оной до самой их конверсии в электрон-позитронные пары?
Аноним 02/11/19 Суб 14:20:43 527066401
>>527046
Потому, что мы видим лишь малую долю спектра фотонов. Рентген в сталь проникает на миллиметры, на этом основана рентгеновская рентгеноскопия. Гамма и на метры может.
Аноним 02/11/19 Суб 14:24:28 527069402
>>527048
Не хуже, чем у темнотряпочников, однако. Я уж о струнных балалаечниках не говорю. Сорок лет уже ничего удобоваримого противопоставить никто не может, одно шельмование.
Аноним 02/11/19 Суб 14:28:11 527070403
>>527062
Самим статфизикам, например. Ни в одном человеческом языке нет короткого описания того, с чем они работают. А конструкции "типа", "похоже" и "вроде согласуется" обычный бинарный человек быдло воспринимает исключительно как слабость и неуверенность.
Аноним 02/11/19 Суб 14:30:45 527072404
>>527070
>Ни в одном человеческом языке нет короткого описания того, с чем они работают.
Как же они с этим тогда работают?
Аноним 02/11/19 Суб 14:36:34 527073405
>>527070
>Самим статфизикам, например.
>быдло не может понять значит статфизикам бомбит!
Пиздец дебил. Быдло в своё время не могло понять, почему земля вокруг солнца вращается, и как движение планет связано с падением яблок на землю.

>>527069
Вся современная наука учитывает результаты экспериментов. Тёмная материя взялась из результата наблюдений. Тёмная энергия взялась из результата наблюдений. Отсутствие эфира взялось из результата наблюдений. Общая теория относительности взялась из результата наблюдений. Теория струн - теоретическая разработка, которую научное сообщество не считает и не будет считать принятой, пока она не объяснит все результаты наблюдений (пока что нет и этого) и не предложит дополнительные эксперименты или предсказания для своей проверки.
Аноним 02/11/19 Суб 14:45:56 527074406
>>527073
а как движение планет влияет на падение яблока?
Аноним 02/11/19 Суб 15:05:30 527080407
Аноним 02/11/19 Суб 15:19:03 527083408
>>527073
>Общая теория относительности взялась из результата наблюдений
Нет.
Аноним 02/11/19 Суб 15:26:24 527085409
>>527083
Доброе утро, давно движение Меркурия наблюдал?
Аноним 02/11/19 Суб 16:07:19 527100410
Если у фотона нету массы, как черные дыры притягивают их? Для гравитации ведь нужна масса.
Аноним 02/11/19 Суб 16:11:53 527103411
>>527100
Наблюдаемая гравитация - это следствие искривления пространства-времени наличием массы (и вообще энергии, масса - форма энергии). Фотон движется по кратчайшему пути, который в искривлённом пространстве оказывается кривым.
Аноним 02/11/19 Суб 16:49:55 527116412
>>527103
А фотон тоже искривляет пространство на чуть-чуть?
Аноним 02/11/19 Суб 17:03:22 527122413
Аноним 02/11/19 Суб 17:06:42 527123414
>>527116
Нет ведь у него нет массы.
Аноним 02/11/19 Суб 17:09:21 527125415
>>527123
Массы покоя нет, но масса в движении все же есть.
Аноним 02/11/19 Суб 18:44:22 527139416
>>527123
Ну дак и пространство искривляется не массой, а тензором энергии-импульса.
Аноним 03/11/19 Вск 00:25:33 527237417
>>527139
Но ведь энергия (не фотона, обычного тела) относительна, зависит от скорости. Получается, что пространство по-разному искривлено для разных наблюдателей?
Аноним 03/11/19 Вск 00:58:27 527241418
Поясните, как работает магнитное поле магнита. В интернете пишут, что его переносят виртуальные фотоны. Что это за виртуальные фотоны такие?
Аноним 03/11/19 Вск 02:24:18 527244419
767676767676.png (23Кб, 970x676)
970x676
Если мы на орьбиту Земли доставим орсбитальную станцию в виде бублика и оставим на орбите так, чтобы бублик вращался не абы как, а вокруг дырки, через которую проходит ось вращения, а сам бублик находится "по экватору". При этом, ось вращения всегда расположения по касательной. То есть, бублик какбы катится по поверхности Земли, её не касаясь и всегда стоит перпендикулярно, как огромная монета на ребре с точки зрения наблюдателя а Земле.
Так вот, очевидно, что из-за того, что чась бублика находится к Земле ближе, а часть - дальше, должна появиться разница во вращательном моменте и бублик должен начать вращаться. А как он будет вращаться? Я понять не могу, в какую сторону закрутит это колесо, вычислительной мощности не хватает
Аноним 03/11/19 Вск 03:03:07 527246420
Анон, пару лет назад видел в передаче на Дискавери, и полгода-год назад обсуждали тут особо интересный метод сбивания всякого говна с орбиты: ядерный заряд на дне километровой шахты заливают водой, сверху ложат болванку, бомбу взрывают и вуаля, болванка достигает чуть ли не релятивистских скоростей
Как такая схема называется? Есть ссылки почитать подробнее?
Аноним 03/11/19 Вск 03:05:41 527247421
>>527125
>но масса в движении все же есть
Нет, нету. Фотон это не частица, это колебание поля наблюдаемое нами как частица, причем безмассовая частица, потому как не взаимодействует с полем Хиггса. Да, мы живем в вакууме который наполнен полями, которые мы наблюдаем как материю.
Аноним 03/11/19 Вск 03:06:52 527248422
Аноним 03/11/19 Вск 03:12:56 527249423
>>527246
>Как такая схема называется?
Идиотская.
Аноним 03/11/19 Вск 03:39:48 527253424
tf.jpg (140Кб, 2328x942)
2328x942
Если есть всякие идеи про терраформирование Марса, вплоть до безумных - уронить астероид, растопить полярную шапку всем ядерным арсеналом Земли, построить орбитальное зеркало и все такое прочее, что все равно толком не поможет, но проекты есть - интересно, а есть ли проекты (раз можно сколь угодно фантазировать) терраформирования Венеры? Есть ли хотя бы теоретический (для Марса озвучены тоже не особо практические) способ сдуть с нее часть атмосферы чтобы стало нормальное давление хотя бы?
Аноним 03/11/19 Вск 09:28:12 527283425
>>527244
Разорвет приливными силами, т к. слишком близко. Гугли предел Роша.
Аноним 03/11/19 Вск 09:29:58 527284426
>>527253
Нужно перевозить атмосферу с венеры на марс, очевидно же.
Аноним 03/11/19 Вск 09:56:04 527286427
>>527253
>способ сдуть с нее часть атмосферы чтобы стало нормальное давление хотя бы?

Масса атмосферы Венеры 4800000000000000000000 кг, что в 93 раза превышает массу всей атмосферы Земли. Способов "сдуть" даже
сотую часть этой массы и при этом без разрушения планеты пока не придумали.
Аноним 03/11/19 Вск 11:08:44 527296428
>>524797 (OP)
Есть ли в космосе молекулы N2O? Может ли атмосфера экзопланет состоят из N2O?
Аноним 03/11/19 Вск 11:41:01 527302429
>>527286
Способ есть и очень простой - сначала нужно сбросить на Венеру Цереру, или 5-6 ледяных астероидов поменьше, чтобы насытить планету водой. Затем нужно вывести микробов-экстремофилов, усваивать СО2 и воду используя энергию солнца. В отсутствие других животных даже если эти бактерии будут делиться раз в сутки, то они экспоненциально размножатся и заселят всю планету за пару месяцев.
В результате мы получем планету с водой и кислородом, заселенную зелеными одноклеточными.
Аноним 03/11/19 Вск 11:53:39 527303430
>>527302
>сбросить на Венеру Цереру
Это разрушит кору планеты и возможно и саму планету.
Аноним 03/11/19 Вск 12:25:46 527305431
Почему геостационарные спутники не падают, если мы берём землю за точку отсчёта?
Аноним 03/11/19 Вск 12:46:01 527312432
>>527305
Они падают, просто не успевают попасть в землю.
Аноним 03/11/19 Вск 12:49:35 527315433
>>527247
>переносит энергию и импульс
>безмассовая частица
Это же надо так не дружить с логикой.
Аноним 03/11/19 Вск 12:52:16 527316434
>>527305
почему самолет не падает если я сижу в салоне и меня взять за точку отсчета? ведь если я точка отсчета, то самолет не двигается, следственно не может лететь. шах и мат
Аноним 03/11/19 Вск 12:53:16 527317435
>>527244
Взгляни на Луну - она вращается синхронно с обращением по орбите. Так же и твой бублик будет вращаться.
Аноним 03/11/19 Вск 12:55:20 527318436
Аноним 03/11/19 Вск 12:55:37 527320437
>>527241
Виртуальные фотоны это такая теоретическая хрень, примерно как теплород раньше был, даже хуже теплорода.
Аноним 03/11/19 Вск 13:04:02 527323438
>>527246
>заливают водой
Зойчем? Чтобы вода сдетонировала?
Аноним 03/11/19 Вск 13:06:17 527325439
>>527318
Масса тесно связана с понятиями энергии и импульса. Масса это характеристика переносчика энергии и импульса. Утверждать что фотон не имеет массы - все равно что говорить энергии и импульсу не нужен переносчик, что они могут существовать сами по себе, а это нарушение логики.
Аноним 03/11/19 Вск 13:50:54 527338440
>>527315
>Это же надо так не дружить с логикой.
Энергия это не масса. Энергия - это амплитуда вектор-потенциала поля. Кто же виноват что это поле не взаимодействует с полем Хиггса? :3
Аноним 03/11/19 Вск 13:56:40 527344441
>>527338
Какого-какого поля?
Аноним 03/11/19 Вск 14:01:07 527347442
>>527344
Скалярное хиггсовское поле.
Аноним 03/11/19 Вск 14:24:25 527355443
>>527312
Но ведь относительно земли у геостационарного спутника нет скорости, он просто висит
Аноним 03/11/19 Вск 14:26:07 527356444
image.png (2594Кб, 1600x1067)
1600x1067
>>527246
>полгода-год назад обсуждали тут особо интересный метод доставки космонавтов на мкс: пороховой заряд на дне пушки, сверху ложат космонавта, порох взрывают и вуаля

Аноним 03/11/19 Вск 14:27:00 527357445
>>527253
как насчет того чтоб просто вывести вид людей, который сможет там жить. базарю это будет быстрее чем сдуть атмосферу
Аноним 03/11/19 Вск 14:28:03 527359446
15671887440150.jpg (176Кб, 460x412)
460x412
>>527355
>относительно земли у геостационарного спутника нет скорости
Аноним 03/11/19 Вск 14:32:59 527362447
image.png (459Кб, 616x410)
616x410
>>527355
Школотрон на пикче решительно не понимает почему его сносит в сторону. Ведь он смотрит на лысого батю а батя на него и их лица неподвижны друг относительно друга. То есть они неподвижны и школотрон как бы не имеет скорости относительно бати. Однако, по странной причине со школотнона слазят штаны и слетели тапки. Сейчас батя отпустит руки и школотрон поймет, что зря сбежал с урока Урины Ивановной, которая преподает физику.
Аноним 03/11/19 Вск 14:33:18 527363448
>>527355
Относительно центра Земли - есть скорость.
Аноним 03/11/19 Вск 14:36:28 527364449
>>527253
Есть целая статья на википедии про терраформирование Венеры. Вообще, это выглядит не сложнее, чем терраформировать Марс. То есть невыполнимо при текущем уровне технологий и не нужно при будущем.
Аноним 03/11/19 Вск 14:39:13 527365450
>>527320
Пиздец. А что тогда реально переносит магнитную силу? Я раньше думал, что обычные фотоны электромагнитного излучения, но оказалось, что нет
Аноним 03/11/19 Вск 14:41:06 527366451
>>527302
Даже если весь углерод вынуть из атмосферы, он покроет поверхность планеты черным слоем, и она будет поглощать слишком много тепла.
Аноним 03/11/19 Вск 14:42:17 527367452
>>527303
тогда не сбросить, а плавно опустить.
Аноним 03/11/19 Вск 14:43:20 527369453
>>527364
А ты попроси церковь квантовых верунов открыть червоточину с венеры на марс, чтобы излишки атмосферы туда утекли.
Аноним 03/11/19 Вск 14:48:32 527372454

>>527365
>А что тогда реально переносит магнитную силу?
Поле.
Аноним 03/11/19 Вск 14:49:21 527374455
>>527367
>плавно опустить.
Каким образом плавно опустить астероид на планету?
Аноним 03/11/19 Вск 14:51:54 527375456
>>527372
Что за поле? Что оно из себя представляет? Почему оно беспрепятственно проходит через пространство и материю?
Аноним 03/11/19 Вск 14:52:41 527376457
>>527374
Никак. При приближении его порвет гравитацией.
Аноним 03/11/19 Вск 15:10:54 527380458
Аноним 03/11/19 Вск 15:13:58 527382459
>>527380
Как ты будешь ломать цереру на части? У нее своя гравитация есть так то
Аноним 03/11/19 Вск 15:14:58 527383460
На Венеру все равно надо будет что то скидывать, чтобы раскрутить ее. Она же медленно крутится.
Аноним 03/11/19 Вск 15:19:25 527386461
>>527375
>Что за поле?
Магнитное
>Что оно из себя представляет?
Что из себя оно представляет мы не знаем. Мы знаем как оно себя проявляет при наблюдении.
>Почему оно беспрепятственно проходит через пространство и материю?
А почему оно не должно? Вся материя состоит из элементарных частиц, а частицы это то, как мы наблюдаем поля, частицы это локальные колебания этих полей. Мир это вакуум наполненный полями.
Аноним 03/11/19 Вск 15:26:05 527387462
>>527382
Уже сказали же, она сама разберётся, когда будет ближе предела Роша.
Аноним 03/11/19 Вск 15:31:09 527390463
>>527386
А как это поле работает? Ну то есть не может же быть, чтобы просто пустота что то делала. У пустоты же не может быть свойств.
Аноним 03/11/19 Вск 15:32:33 527391464
>>527387
Частично разберется, но большая часть просто свалится.
Либо, если пытаться опускать по спирали, она развалится вся, но большая часть не упадет, а останется на орбите в виде кольца.
Аноним 03/11/19 Вск 15:38:38 527396465
>>527390
>А как это поле работает?
Послушай внимательно ответ мистера Фейнмана.
>>527248
Аноним 03/11/19 Вск 15:42:16 527398466
>>527391
Ну так в этом и смысл, опустить на низкую орбиту, чтобы развалилась в кольцо, и дальше опускать куски.
Аноним 03/11/19 Вск 15:43:54 527399467
>>527391
Или рассчитать траекторию таким образом, чтобы основной кусок задел атмосферу, там развалился дальше и упал с аэроторможением.
Аноним 03/11/19 Вск 15:44:16 527400468
>>527365
>А что тогда реально переносит магнитную силу?
Очевидно что. Материя. Из материи состоят все поля, все частицы. Даже в вакууме есть материя, из которой строятся гравитационное поле и остальные поля.
Аноним 03/11/19 Вск 15:48:00 527401469
>>527400
В вакууме же плотность материи 1 атом водорода на кубический километр, или что то типа того. Как этот атом будет переносить поле?
Аноним 03/11/19 Вск 15:48:43 527402470
>>527398
Оно неконтролируемо развалится. А куски ты будешь миллион лет скидывать по одному.
Аноним 03/11/19 Вск 15:50:19 527403471
CYBERPUNK.mp4 (16752Кб, 1920x960, 00:00:31)
1920x960
Аноним 03/11/19 Вск 15:51:50 527404472
>>527401
Водород это вещество, а материя это более широкое понятие, включающее в себя и вещество и поля и всё остальное, что может находится в пространстве.
Аноним 03/11/19 Вск 15:52:28 527405473
>>527403
Зачем такое будущее нужно? Задыхаться в обосраном городе, не видя солнечного света.
Аноним 03/11/19 Вск 15:53:23 527406474
>>527404
То есть поле - это что то типа неосязаемого атома?
Аноним 03/11/19 Вск 15:54:10 527407475
>>527406
А почему атом осязаемый?
Аноним 03/11/19 Вск 15:56:04 527408476
>>527405
Двачую. В будущем городов не будет, все будут жить в частных домиках окруженных значительной территорией. До ближайших соседей надо будет идти пол километра.
Аноним 03/11/19 Вск 15:57:31 527409477
>>527407
Из-за электронной оболочки.
Аноним 03/11/19 Вск 15:57:35 527410478
>>527406
Поле физическое это некий материальный объект, который с некоторой степени точности можно описать математическим полем.
Аноним 03/11/19 Вск 15:57:53 527411479
>>527408
>идти пол километра
Только если поставишь мод на отключение фасттревела.
Аноним 03/11/19 Вск 15:58:12 527412480
>>527409
Ну а как ты её осязаешь?
Аноним 03/11/19 Вск 15:59:45 527414481
>>527411
Да, в будущем же будут всякие ховерборды, быстро перемещающие куда надо.
Аноним 03/11/19 Вск 16:00:43 527415482
>>527410
То есть он материальный, этот кусок кругляшей из одного полюса магнита в другой, что то типа луковицы, но невидимый. Так? Это очень странно конечно. Какая то ебака проходит все насквозь, но при это невидима, но при этом еще и материальна. Из частиц не состоит, только из пустоты.
Аноним 03/11/19 Вск 16:01:06 527416483
>>527414
Ну да, хоть ховерборды, хоть фтл, главное чтобы электричество в ф*зическом мире не отключили.
Аноним 03/11/19 Вск 16:01:33 527417484
>>527412
Электронные оболочки поверхности моего пальца взаимодействуют с электронной оболочкой атома или атомов.
Аноним 03/11/19 Вск 16:03:11 527419485
>>527417
Ну вот и твой ответ. Все твои осязания - это взаимодействия полей.
Аноним 03/11/19 Вск 16:03:47 527420486
>>527416
Электричество будет слишком старой технологией. Только ностальгирующие по древним векам ретрограды будут пользоваться электрическими приборами.
Аноним 03/11/19 Вск 16:05:33 527422487
>>527419
То есть человек состоит из невидимой хуйни? Ппц. Так и в ауры всякие поверить можно.
Аноним 03/11/19 Вск 16:06:56 527423488
>>527406
>То есть поле - это что то типа
Нет аналогий в известном тебе мире. Сравнить не с чем.
Аноним 03/11/19 Вск 16:07:16 527424489
>>527420
Ностальгирующие ретрограды будут жить в древних веках, в замках и дворцах с гетерами. Люди прогрессивные будут рассекать на звездолётах по галактикам и ебать инопланетянок. Некоторые действительно откажутся от электричества, будут пользоваться магией, пулять фаерболы в драконов.
Аноним 03/11/19 Вск 16:07:55 527425490
>>527422
>невидимой
Видение - это взаимодействие фотонов с электронами в твоём глазу.
Аноним 03/11/19 Вск 16:08:27 527426491
>>527410
>Поле физическое это некий материальный объект
Тогда встает вопрос что такое материальный объект. Вдеь поля есть везде.
Аноним 03/11/19 Вск 16:09:28 527427492
>>527415
Там нет кругляшей и всяких силовых линий, там есть материя, по которой передаются взаимодействия. Что такое материя, из чего она состоит, как именно устроена - мы пока не знаем. Но можно догадываться, строить всякие теории, авось верную придумаем или близкую к верной. Материя это не пустота, это наоборот.
Аноним 03/11/19 Вск 16:11:12 527428493
>>527427
Тогда космического вакуума не су
Аноним 03/11/19 Вск 16:11:42 527429494
>>527428
ществует, т.к. все заполнено полями.
Аноним 03/11/19 Вск 16:13:55 527431495
>>527427
> мы пока не знаем
То есть может быть так, что магнитное поле - это какие то мельчайшие частицы, про которые мы пока не знаем, передвигающиеся по луковичным орбитам и толкающие вещество?
Аноним 03/11/19 Вск 16:14:37 527432496
>>527415
>То есть он материальный, этот кусок кругляшей из одного полюса магнита в другой, что то типа луковицы, но невидимый

Поля для тебя и меня проявляются по-разному, мы с тобой поля наблюдаем как элементарные частицы, как нейтроны, электроны, протоны. Колебания полей. Представь себе Вселенную как океан и как будто огромная морская гладь, по которой несутся волны и создают рельеф. Наивная аналогия конечно, но раз ты их хочешь, то она ничем не хуже и не лучше других.
Аноним 03/11/19 Вск 16:16:04 527433497
>>527427
>Что такое материя, из чего она состоит, как именно устроена - мы пока не знаем

атом -- атомное ядро -- нуклоны -- кварки.
Аноним 03/11/19 Вск 16:16:27 527434498
>>527423
Есть. Математически в виде поля можно описать и макрообъекты, которые хорошо известны. Например, обычный воздушный ветер можно описать в виде поля. Можно назвать ветер физическим полем. Носитель этого поля - воздух.
Аноним 03/11/19 Вск 16:16:40 527435499
>>527432
Хочешь сказать, что вся материя - это поля? Только одни поля тверже и непрозрачнее других?
Аноним 03/11/19 Вск 16:16:53 527436500
>>527431
>То есть может быть так, что магнитное поле - это какие то мельчайшие частицы,
Нет, не частицы. Это поле. Что такое поле? Ты не хочешь математических формул, а хочешь аналогии из окружающего тебя мира?
Аноним 03/11/19 Вск 16:17:24 527437501
>>527433
Поясни, почему кварки нельзя оторвать друг от друга?
Аноним 03/11/19 Вск 16:17:43 527438502
>>527435
>Хочешь сказать, что вся материя - это поля?
Именно так.
Аноним 03/11/19 Вск 16:17:52 527439503
>>527431
Да, ты можешь придумать такую теорию.
Аноним 03/11/19 Вск 16:18:27 527440504
>>527436
Да, потому что математика не объясняет, почему поле действует на объекты. Она объясняет только, как действует.
Аноним 03/11/19 Вск 16:19:58 527441505
>>527438
А если взять человека, то как так случилось, что поле-человек имеет такую сложную форму?
Аноним 03/11/19 Вск 16:20:16 527442506
>>527437
>Поясни, почему кварки нельзя оторвать друг от друга?
Почему? Потому что так устроен мир.
Аноним 03/11/19 Вск 16:21:15 527443507
>>527442
Это не ответ на вопрос, а по сути повтор вопроса.
Аноним 03/11/19 Вск 16:31:34 527445508
>>527438
А вот электронная оболочка атома она же не шарообразная, а меняется в зависимости от каких то условий. А можно то же самое сказать про оболочку протона? Кварков же по идее три, а из трех частиц никак шаровидный объект не построишь. Кварки тоже крутятся по каким то орбиталям, как электроны?
Аноним 03/11/19 Вск 16:39:18 527447509
>>527359
>>527362
>>527363
Я понимаю, что мой ход мыслей ошибочен, но я не понимаю, почему, и что надо учесть, когда за систему отчёта принимают Землю.
Аноним 03/11/19 Вск 16:47:29 527448510
>>527447
Ничего не надо учитывать, относительно земли (центра земли) он вращается. А поверхность земли, относительно которой спутник покоится, сама вращается, то есть не является инерциальной системой отсчёта.
Аноним 03/11/19 Вск 17:01:23 527450511
>>527422
аура - это часть тела которая состоит из темной материи
Аноним 03/11/19 Вск 17:04:12 527452512
>>527450
Ты думаешь, в человеке есть темная материя? А в Земле она есть? А в Солнце?
Аноним 03/11/19 Вск 17:06:26 527453513
>>527452
Никто не знает. Если тёмная материя состоит из вимпов, то есть наверняка.
Аноним 03/11/19 Вск 17:10:06 527454514
>>527453
Но тогда бы Солнце или Земля были бы тяжелее, чем получается по рассчетам
Аноним 03/11/19 Вск 17:15:33 527455515
>>527437
Почему это, еще как можно. Только энергии надо будет столько затратить, что хватит на материализацию новой пары кварк-антикварк, и будет выглядеть так, как будто ты ебанул по частице, а из нее вылетел мезон, а не одиночный кварк.
Аноним 03/11/19 Вск 17:17:52 527456516
>>527455
Я это и имел в виду.
А нельзя как то по другому манипулировать кварками, без накачки таким большим количеством энергии?
Аноним 03/11/19 Вск 17:18:24 527457517
>>527455
А энергия - это тоже поле?
Аноним 03/11/19 Вск 17:26:11 527458518
>>527456
Нельзя. Цветное взаимодействие очень сильное и не ослабевает с расстоянием, так что его невозможно разорвать, не заткнув неспаренные кварки подходящим цветом.
Аноним 03/11/19 Вск 17:27:26 527459519
>>527458
А если один кварк зажать пассатижами и потянуть?
Аноним 03/11/19 Вск 17:29:12 527460520
>>527458
В смысле не ослабевает с расстоянием? Такое возможно? Это же получается 100% кпд?
Аноним 03/11/19 Вск 17:31:52 527461521
А что будет, если прицельно хуйнуть одним электроном в другой?
Аноним 03/11/19 Вск 17:33:14 527462522
А есть ли у времени свое поле. Или свои частицы?
Аноним 03/11/19 Вск 17:34:42 527463523
>>527438
А можно ли сказать, что вся Вселенная состоит из полей?
Аноним 03/11/19 Вск 17:39:06 527464524
>>527454
Нет, они были бы такие, какие получаются по измерениям. А вот их плотность была бы немного другой. Но, думаю, разница была бы несущественной.
Аноним 03/11/19 Вск 17:43:20 527465525
>>527464
Я читал, что темная материя расположена за пределами галактики в виде сферического гало или что то типа того.
Аноним 03/11/19 Вск 17:47:03 527466526
>>527465
Если она состоит из вимпов, ничто не помешает ей образовывать гало вокруг любых тел. И не за пределами, а и внутри тоже.
Аноним 03/11/19 Вск 17:47:21 527467527
Аноним 03/11/19 Вск 17:53:42 527468528
>>527467
А что тогда энергия? Параметр поля?
Аноним 03/11/19 Вск 18:15:31 527469529
>>527464
>>527466

Я хотел написать, что тогда бы дальние планеты в Солнечной системе вращались быстрее, чем они движутся сейчас. К центру бы их притягивала не только масса Солнца, но и масса гало темной материи, а ближние планеты — только масса Солнца, и можно бы было проверить экспериментально, есть ли гало вимпов или нет. Потом, правда, оказалось, что я не первый такой умный, но эффект настолько мал, что достоверно измерить его с современными инструментами пока нельзя.
Аноним 03/11/19 Вск 18:30:26 527472530
Аноним 03/11/19 Вск 18:31:33 527473531
>>527472
Ну это я образно.
Но это не нарушает законы термодинамики, сохранения энергии и прочия прочия?
Аноним 03/11/19 Вск 18:43:29 527474532
>>527247
>причем безмассовая частица, потому как не взаимодействует с полем Хиггса
А бозон хиггса это искомый гравитон? Нет? Тогда при чем тут он?

Массы покоя у фотонов нет, никто не спорит. Но масса-энергия - есть. Гипотетически очень много света даже может сколлапсировать в черную дыру под своей (света) массой.

https://ru.m.wikipedia.org/wiki/Кугельблиц
Аноним 03/11/19 Вск 18:45:39 527475533
А пространство - это тоже поле?
Аноним 03/11/19 Вск 18:51:59 527478534
>>527474
А бозон хиггса это искомый гравитон? Нет? Тогда при чем тут он?
При том, что поле Хиггса обеспечивает наличие массы, а не гравитации.

>Массы покоя у фотонов нет
А никакой другой и не бывает, если не пользоваться устаревшей (или школьной) формулировкой эквивалентности массы и энергии.

>сколлапсировать в черную дыру под своей (света)
Энергией.
Аноним 03/11/19 Вск 18:53:28 527479535
>>527478
А коллапсирует поле, получается? То есть становится бесконечным?
Аноним 03/11/19 Вск 19:33:48 527486536
Аноним 03/11/19 Вск 22:55:52 527507537
>>527383


Там все равно жарко и газы
Как жить то?
Аноним 03/11/19 Вск 23:52:07 527514538
Аноним 04/11/19 Пнд 00:27:16 527520539
Аноним 04/11/19 Пнд 02:03:15 527544540
>>527457
>А энергия - это тоже поле?
Что бы не заморачиваться, просто рассматривай это как классические амплитуды поля.
Аноним 04/11/19 Пнд 02:03:44 527545541
>>527463
>А можно ли сказать, что вся Вселенная состоит из полей
Да. Наверно можно.
Аноним 04/11/19 Пнд 02:08:07 527546542
>>527474
>А бозон хиггса это искомый гравитон?
Нет. А какюу связь ты видишь между спонтанным нарушением электрослабой симметрии и гравитацией?
>Тогда при чем тут он?
При том, что именно скалярное хиггсовское поле "определяет" массы того что мы называем элементарные частицы.
Аноним 04/11/19 Пнд 02:11:54 527549543
>>527479
>А коллапсирует поле, получается? То есть становится бесконечным?

Собственно поле и так бечконечно. :3
Аноним 04/11/19 Пнд 02:23:58 527553544
Аноним 04/11/19 Пнд 02:30:31 527555545
Какой закон запрещает путешествовать в прошлое?

Причинность.

Но если я путешествую в прошлое, то я там появляюсь например через червоточину, то моя причина это будущее. Ну и какой парадокс. Вот прилетел я в будущее, убил там своего деда, и живу дальше, никаких проблем. Я то уже родился. Сначала причина, потом следствие.

Парадокс убийства деда решён.

Так какие законы запрещают путешествие в прошлое?
Аноним 04/11/19 Пнд 02:31:40 527556546
>>527555
>Вот прилетел я в будущее, убил там своего деда, и живу дальше, никаких проблем. Я то уже родился. Сначала причина, потом следствие.
>Вот прилетел я в ПРОШЛОЕ
fix
Аноним 04/11/19 Пнд 02:42:21 527557547
>>527555
>Так какие законы запрещают путешествие в прошлое?

Есть гипотеза, что для полей время не существует и к примеру все электроны являются одним электроном, находящимся одновременно в разных точках пространства-времени. :3

https://en.wikipedia.org/wiki/One-electron_universe
Аноним 04/11/19 Пнд 09:46:26 527590548
>>527555
Ты появляешься в прошлом, хотя в этот момент даже твой батя не родился. Это нарушение принципа причинности. Причина - секс твоего бати, следствие - твое рождение. А у тебя следствие появляется раньше причины.
Аноним 04/11/19 Пнд 12:46:00 527618549
>>527408
В треде солярианец?
Аноним 04/11/19 Пнд 15:25:58 527649550
>>527557
Слышал, но время существует для людей состоящих из электронов
Аноним 04/11/19 Пнд 15:28:44 527654551
>>527590
Но я там же появляюсь не волшебным образом, а причинно следственным образом через червоточину.
Мое следствие будущее. Батя не родился ну и что. Еще родится. А потом родится копия меня и будет два меня и это изменит мир тем что два меня. Другой мир из которого я выбрался будет жить с 0 меня (после того как сьебал). То есть будет просто несколько версий мира. Кто сказал что мир один?
Аноним 04/11/19 Пнд 15:50:15 527667552
>>527649
>состоящих из одного электрона
Не благодари.
Аноним 04/11/19 Пнд 15:58:23 527671553
>>527654
Червоточина и есть волшебный способ.
Аноним 04/11/19 Пнд 16:00:12 527672554
>>527667
Тогда получается, что один электрон путешествует по гигантской вселенной с невообразимой скоростью на порядки выше скорости света. Это нарушение законов физики.
Аноним 04/11/19 Пнд 21:39:25 527811555
ZpWxtzWYS8Up5Pu[...].jpg (100Кб, 600x667)
600x667
Аноним 05/11/19 Втр 00:37:54 527866556
>>527402
Нас семь миллиардов на Земле. Каждый возьмет по куску и мы справимся.
Аноним 09/11/19 Суб 15:22:56 529181557
>>526854
>Если марс лежит у тебя на столе, то ближайшая звезда в другом микрорайоне, а ближайшая галактика в другом городе.
Так блэт! Я понимаю, что это наглядная зарисовка в бытовых декорациях. И что использована "логарифмическая шкала". Но мой внутренний зануда кипит настолько, что спустя неделю не может проигнорировать такое "допущение" в уже утонувшем треде. Ибо вижу какую-то вредную недооценку в масштабе расстояний меж галактиками. Если Марс на столе (50см), то звезда в другом городе - несколько десятков километров. А галактика как раз на Марсе.
Аноним 10/11/19 Вск 04:54:43 529317558
image.png (332Кб, 1024x1024)
1024x1024
Аноним 10/11/19 Вск 10:00:19 529334559
>>529317
Всегда пожалуйста, обращайтесь!
Ваш буквоед
Настройки X
Ответить в тред X
15000 [S]
Макс объем: 40Mб, макс кол-во файлов: 4
Кликни/брось файл/ctrl-v
Стикеры X
Избранное / Топ тредов